Science topics: CosmologyMass
Science topic

Mass - Science topic

Explore the latest questions and answers in Mass, and find Mass experts.
Questions related to Mass
  • asked a question related to Mass
Question
3 answers
In plants
Relevant answer
Answer
If you mean the density of the root mass in the soil or growing medium, the density of the root mass (RMD) is expressed as milligrams of dry roots per unit volume of the soil where the roots are located (mg Root Dry Weight/cm3 Sampled Soil Volume.
But the density of root is expressed as mg of Dry Root/Sampled Root Volume.
  • asked a question related to Mass
Question
8 answers
Is the energy of universe conserved while its entropy is increasing and what would happen if two black holes collided that are both the same size and mass?
Relevant answer
Answer
Energy of universe is a sum of different forms of energy and that sum remains constant, but proportions could vary. At the occurrence of every natural process the entropy of the universe increases while its energy remains constant. However as the entropy (energy not available for useful work) increases then the Gibbs free energy (energy available for useful work) decrease. The increase of entropy, the dissipation of matter and energy goes on until our universe becomes so infinitely disordered that entropy can no longer increase and events come to an end. This is called the heat death of the universe. Some say that, because things cannot get any worse, nothing happens at all. Energy of Universe is conserved but entropy of Universe always increases during any natural process. Energy conservation in an expanding universe refers to the idea that the total energy of the universe remains constant as it expands. This means that while the distribution of energy may change, the overall amount of energy in the universe does not. However, if we consider a simplified scenario where two identical black holes (with equal mass and no other complicating factors) collide: Formation of a Larger Black Hole: When the two black holes merge, they create a single, more massive black hole. If two black holes with the same mass collide head on at the same speed, the total momentum is zero. Real black holes are, in fact, much larger than that because they normally contain much more mass. At a minimum, black holes have a few times the mass of our Sun. But let's imagine that a black hole with the mass of the Earth can exist for a moment. It would indeed be less than 1 inch across.
  • asked a question related to Mass
Question
2 answers
What happens to gravitational potential energy as distance increases and what happens to gravitational attraction of mass or distance changes?
Relevant answer
Answer
Respected Sir
  • Gravitational Potential Energy (PE): Increases as distance increases (moves away from a massive object) and decreases as distance decreases (moves closer to a massive object).
  • Gravitational Attraction: Decreases as distance increases (objects move farther apart) and increases as distance decreases (objects move closer together). This relationship is described by the inverse square law of gravity.
  • asked a question related to Mass
Question
8 answers
Gravity is often considered a pseudo-force that is associated with the geometric curvature of spacetime. The electromagnetic force is considered a gauge force transferred by virtual photon messenger particles. Gauge theories are field theories that have the property of gauge invariance. These two mechanisms appear to be completely different.
However, this is a discussion question, and I will attempt to prove these two forces are closely related. The referenced preprint below presents a model of the universe based entirely on waves. An electron and other fermions are modeled as rotating soliton waves that exist in the sonic medium of oscillating spacetime. This model predicts that the electron’s electrical charge and its gravitational curvature are both the result of an electron’s wave properties interacting with the medium of oscillating spacetime.
When these forces are viewed as wave effects, predictions are made that these two forces should merge at the wave limit. The maximum charge that a wave-based particle can produce is Planck charge (qp) and the maximum mass a wave-based particle can produce is Planck mass (mp). The prediction is that the electrostatic force between two Planck charges (Fqp) equals the gravitational force magnitude between two Planck masses (FGp). At arbitrary separation distance r, the force magnitude is Fqp= FGp = ħc/r2. This merging of these forces at the Planck limit can be turned into equations that show that the forces are also closely related even at the level of two electrons or between any other particles.
What do you think? Are these forces fundamentally different? Is gravity even a force?
Relevant answer
Answer
No, I'm not referring to whether it's ``insignificant''; I'm saying that classical gravity doesn't make sense when quantum effects are relevant. The mathemaical expressions are wrong (for electromagneism) and meaningless (for gravity) when quantum effects become relevant.
  • asked a question related to Mass
Question
5 answers
How do unequal heating and the movement of air at the equator and at the poles produce global wind patterns and what does the unequal heating of air masses cause?
Relevant answer
Answer
Unequal heating and the movement of air at the equator and poles produce global wind patterns as follows:
  • Equator (Tropical Regions):Intense solar heating at the equator warms the air, causing it to rise and create low pressure. Rising warm air moves towards the poles at high altitudes, then cools and sinks back towards the surface at around 30 degrees latitude (forming subtropical high-pressure areas). This descending air creates stable, dry conditions and drives surface winds towards the equator (trade winds).
  • Poles (Polar Regions):Cold polar air sinks and moves towards lower latitudes along the surface, creating polar easterlies. This air meets warmer air from the mid-latitudes, where it rises and forms low-pressure areas, driving the westerlies.
Unequal Heating of Air Masses:
  • Unequal heating causes variations in air temperature and pressure.
  • This leads to the formation of pressure gradients and wind patterns as air moves from areas of high pressure to low pressure.
  • Wind patterns and air masses play a key role in shaping weather systems and climate around the globe.
  • asked a question related to Mass
Question
5 answers
In my theory, the speed of gravitational waves could be 76 m/sec greater than the speed of photons. If what I'm saying could be true (i.e. experimentally verifiable), then it would be proof that the photon has a non-zero mass!!
Relevant answer
Answer
Science is following standard modeling theory of atom. This theory still is a theory. When it comes to proton/neutron and electron becomes a theory over another theory, because up to date nothing of inside of atom, never been seen such as proton/neutron..etc.
In other hand, science confused between light and speed of sunlight.
Furthermore, science has no clue about gravity.
I'm totally lost with this question.
  • asked a question related to Mass
Question
2 answers
My objective is to find the binding affinity of divalent metal ions with polyacrylic acid (PAA) by Isothermal Titration Calorimetry. In this experiment, I need to prepare 2mM of PAA. The 2mM should be in monomer concentration terms. So, How do I calculate how much mass of polyacrylic acid do I need to measure if the average molecular weight(Mw) of choosen polyacrylic acid is 12000. If someone knows, please tell me in detail with mass calculation strategies.
Relevant answer
Answer
The answer is perfect for beginners to learn about it Thank you so much for your nice reply.
  • asked a question related to Mass
Question
3 answers
Dear Friends, Grand Success. The 119th finding in TOU (Theory of Universality) is that G at the periphery of Cosmos is theoretically calculated as G' = 1.82266 x 10^16 terrestrial SI units as per Lorentz transformations; which agrees with the G' calculated as 1.81423 x 10^16 terrestrial SI units as per my Theory. Both agree. This is the theoretical proof that all the constants change as we move up and down from the spherical shell of our OU (27.6 Billion light years thickness) and from a radius of 1.64682 Trillion Trillion light years from the CCBH ( Central Cosmic Black Hole ). For example, as per TOU, G = (4 c^3. lambda_p^2. m_p) / (N_a^2/3. h), where, c is speed of light, lambda_p is the Compton wavelength of proton, m_p is the mass of proton , N_a is Avogadro constant and h is Planck constant. The above formula for G yields a value of 6.66889 x 10^-11 SI units , which perfectly agrees with experimental value of G. Now, as space-time changes when we move across the Cosmos radially, c changes, mass changes with speed ( so does the Compton wavelength). As mass changes with speed Avogadro constant also changes. Finally, h = lmc, and all the three change, so does h. This is because the space-time created for each shell from the CCBH is different. To be published in the next Annexure.
Please, join:
Relevant answer
Answer
Brain doesn't exist. "Brain" is just an idea in consciousness. See my paper "How Self-Reference Builds the World": https://philpeople.org/profiles/cosmin-visan
  • asked a question related to Mass
Question
2 answers
i m post mais idea in youtube beacause in brazail all institutes are comunist. and not have free minds .
maybe mass exist only because universe stay in aceleration
see video in diego silva lemelle chanel in youtube.
link in
Relevant answer
Answer
Brain doesn't exist. "Brain" is just an idea in consciousness. See my paper "How Self-Reference Builds the World": https://philpeople.org/profiles/cosmin-visan
  • asked a question related to Mass
Question
1 answer
I'm keen to view data on the body masses of people (and/or heights), disaggregated by sex, representing a good diversity of human populations, based on large sample sizes and including a measure of spread/error around the means.
Is anyone aware of such a resource? Thank you.
Relevant answer
Answer
Hi Lewis,
There are several databases available online for researchers, which include weight and height. I would try to look at the SHARE datasets (https://share-eric.eu/), as SHARE is a longitudinal study which has been collected throughout the past decades from older adults in many European countries, and has such information.
Good luck!
  • asked a question related to Mass
Question
2 answers
I am expressing a membrane protein (about 15 kDa) in BL21(DE3) cells. The expression is good, and the yield is decent, but each time I checked the mass with a TOF mass spec, the mass I get is nothing close to the expected value for an isotopically labeled protein, especially with 15N. When I introduced 13C glucose, I saw an increase in mass, which shows the incorporation of 13C. This is in contrast with the 15N labeling. I've tried increasing the concentration of 15NH4Cl and eliminating rich broth, but nothing has changed thus far. Does anyone have an idea how to go around this problem? Thank you
Relevant answer
Answer
Thank you so much for your response! My minimal media composition for a 500 mL culture is as follows:
50 mL of 10x sterile M9 salt (66 g Na2HPO4, 30 g KH2PO4, 5 g NaCl; buffered to pH 7.4 )
2g 13C glucose
1g NH4Cl
50 uL (5mg/mL) thiamin
200 uL (2.5 mg/mL) biotin
2.5 mL (1M) MgSO4
5 mL uniformly labeled 13C15N bioexpress
10 uL (5000x) studier trace metals
450 mL autoclaved water
ampicillin
I reviewed the protocol in the link, and it seems very similar to what I'm using; it has also worked well for other proteins I've expressed. I don't know what I'm missing with this one. Thanks for sharing!
  • asked a question related to Mass
Question
2 answers
How are warming ocean temperatures impacting fish and other marine mammals and difference between ocean currents and water mass?
Relevant answer
Answer
Hi, small opening remark, fish are not 'other marine mammals'. Fish and other gill breathing animals are strongly affected by water temperature, irrespective of the origin. First of all, fish, except big tuna, have environment temperature, not a fixed one around 36°C like mammals. The proteins of fish adapted to high latitudes function best in cold or cool waters, where disolved oxygen tends to be much higher than in warm, tropical waters.
As gas exchange in water breathing animals is much more energy intensive than air breathing, fish must expend a large part of their energy budget on breathing. Moreover, gills are surfaces, while the body of fish will grow in volume. At some point, the surface limitation of gas exchange through the gills will not allow fish to transform food into own body cells because of lack of oxygen supply. This is why fish species in the warmer part of their distribution range attain a lower maximum size than in the cooler part of their distribution range. It has also been observed, that e.g. salmon can grow a bit more after spawning (spawning reduces body weight requiring oxygen supply). This gill-oxygen limitation theory (GOLT) proposed by Pauly also explains why we observe polewards migrations of fish species as the ocean surface waters warm. They want to remain within their preferred temperature range - if they can. It means also that tropical regions loose species without replacement because it's becoming too warm (oxygen too low, even without overfishing), while temperate and polar regions gain, at least temporarily. But cold adapted animals may be at risk, e.g. mass mortality of snow crabs in the rapidly warming Arctic. Air breathing marine mammals are not affected by this phenomenon, except indirectly in that they prey species may move as a result of a warming ocean.
  • asked a question related to Mass
Question
63 answers
If anything is physically existent, it can well be considered (1) as matter and energy, because these two are inter-convertible, and (2) as either matter or energy, since these are the two most basic states of physical existents.
But the mass of matter-energy is considered as a quantity, and energy too is considered as a quantity. In these cases, the former manner of considering matter and energy as physical existents is kept away from consideration. Instead, the circumstantially possible measure of matter-energy together is taken as mass, and the circumstantially possible measure of energy alone is considered as energy. Thus, mass and energy may be considered as a pair of quantities too.
How to differentiate these two aspects of doing physical science? How to reconcile them? Note that existent matter and energy as such are not separately and respectively being treated to correspond to the quantitative concepts of mass and energy. We have, as a result, many confusions in physics and in the philosophy of physics. Famously, the difficulty to define mass and energy as quantities might issue from the above discrepancy.
I invite your well-considered viewpoints. Merely holding that physics is such and that we need not ask such questions at all -- such is not the attitude from which the above questions are asked here. Do we have fundamental solutions for these questions?
Raphael Neelamkavil
Relevant answer
  • asked a question related to Mass
Question
4 answers
Everyone knows simple classical model of dielectric susceptibility called Lorentz oscillator model, in other words, spring mass model. But I can't find it's all parameter values for different atoms: resonance (spontaneous) frequency - ω0 = \sqrt{k/m}, 2) effective mass of electron - m, 3) damping coefficient - γ = 1/τ, τ - relaxation/scattering time. If there are experimenters or specialists from relating fields which have knowledge of handbooks, databases, catalogs or any writings for experementers, of values of ω0, m, γ, plasma frequency ωp, real (refractive index or relative permittivity) and imaginary parts of susceptibilities \chi(1), please provide information on them.
Related question is the following: Has the Lorentz model expanded for diatomic, triatomic, ... multiatomic molecules? Is it useful for theoretical description of susceptibilities for multiatomic molecules? Why yes or why not? Please provide relevant information.
Thank you in advance...
Relevant answer
Answer
I PM'd you Thomas Mayerhöfer .
  • asked a question related to Mass
Question
3 answers
according to Newton gravity and GR a star with mass exceeding the Chandrasekhar limit collapses to a point and mass density becomes infinite, what is nonsense and shows the limit of especially GR.. A new formulation of W.Greiner an collaborators has been given, which avoids this problem.
please comment.
Relevant answer
Answer
Greiner's paper is certainly not the solution. nevertheless it s a very interesting ansatz.
  • asked a question related to Mass
Question
1 answer
Hi,
 I'm using Brooks mass flow controller 5851E with Brooks control and readout unit 0152E. The differential pressure is set at 15 psi. Even though the set point is zero at the control unit, the mass flow controller lets through the gas. I cannot completely stop the gas flow, it still shows 2.5% on the control unit when the set point is at 0.
Relevant answer
Answer
Interestingly enough, I have the same problem but with another model of Brooks thermal mass flow controller.
The only way I could completely stop the gas flow is by closing the valve before the controller.
Also, to be fair, Brooks specifies in the manual that the controller readings can be trusted only when the gas flow rate is bigger than 10% of the set point mass flow rate.
  • asked a question related to Mass
Question
4 answers
Gravitational time dilation and time dilation for a moving object are fundamental concepts in the theory of relativity, expressing the equivalence between density and distance traveled in time. In this context, density can be defined as a length in space-time, representing the concentration of mass and energy within a given region of the universe.
Gravitational time dilation occurs in the presence of a gravitational field, where time is experienced differently depending on the strength of the gravitational field. This phenomenon is a direct consequence of Einstein's general theory of relativity, which posits that massive objects cause a curvature in space-time. As a result, time passes more slowly in regions of stronger gravitational fields, such as near massive celestial bodies like stars or black holes.
On the other hand, time dilation for a moving object, also known as relativistic time dilation, arises from the principle that the speed of light is constant for all observers. When an object moves at a significant fraction of the speed of light, time for that object appears to pass more slowly from the perspective of a stationary observer. This effect becomes increasingly pronounced as the object's velocity approaches the speed of light, leading to significant differences in the passage of time between the moving object and the stationary observer.
The equivalence between density and distance traveled in time is a profound insight into the nature of space-time and the interconnectedness of mass, energy, and the passage of time. It suggests that the concentration of mass and energy within a given region of space-time directly influences the experience of time within that region. Furthermore, it underscores the intricate relationship between gravitational forces, relative motion, and the fundamental fabric of the universe.
In summary, gravitational time dilation and time dilation for a moving object reveal the intricate interplay between density and distance traveled in time within the framework of Einstein's theory of relativity. These concepts have profound implications for our understanding of the nature of space-time and the fundamental laws governing the behavior of mass, energy, and the passage of time.
Relevant answer
Answer
Greeting
Thank you for your guidance and valuable reply
You are right.
But the problem that has troubled my mind is that no object falls in the gravitational field without the passage of time.
Can events be reviewed over time?
If we consider time as a true Euclidean dimension, which is imaginary from the point of view of the three-dimensional world, the problem arises that objects in the real dimension of time have momentum, mass, speed, etc.
Imagine an expanding ball. The real dimension z is imaginary and arrow-like from the perspective of beings on the surface of the sphere. Any movement on the surface of the sphere can be related to the eccentricity of the ellipse. The strange point of this problem is that from the point of view of two-dimensional beings, the z-dimension and the time dimension are observed in the same axes.
Can the existence of two types of equivalent mass be the existence of two types of behaviour of matter in space and time!!!
Or does the stress of space and time cause the existence of two types of mass?
Velocity in space is in terms of length in time
Can speed in the time dimension be defined as a length in space?
Is it possible to express mass over time with length in the real dimension of time?
Do space-time geodesics determine the direction of movement in the time dimension? like the direction of mass movement in the field of gravitational!!!!
Density creates heterogeneity in the structure of space and time...
Does the eccentricity of the ellipse in one dimension cause the eccentricity of the ellipse from the perspective of the observer in other dimensions?
Sincerely
Mousavi
  • asked a question related to Mass
Question
1 answer
Here I present my findings that the solar system mirrors atomic systems and has a quantum mechanical solution to the Schrödinger wave equation based on a base unit of 1 second. I further suggest that this solution for our spectral class G2V star the Sun, may indicate that life is a uniform cosmic unfolding throughout the Universe. In order to test whether this works for many G2V star like our Sun we would need to detect Earth sized planets around such other such stars, to see if my Planck constant works for other planetary systems, which I show is connected to the my theory for the radius of a proton in terms the Natural constants such as the gravitational constant for gravity and the Planck constant for atoms. However the first Earth-sized planets have been detected in the habitable zones of stars, and interestingly there are 4 planets in a red dwarf spectral class M star in the habitable zone of a star designated TOI 700 with complete data and fairly small margins of error. Two of them are Earth-sized, but the star is not spectral class G2V like our Sun. However, before that we detected an Earth-like planet in the habitable zone of a G2V star like our Sun, but it was not Earth-sized, but rather a super-Earth, which is to say it was much more massive than the Earth, estimated 5 times more massive. And this figure for the mass was an estimate, where for our Earth-sized planets around an M star are well determined. The star for the super-Earth is called Kepler-452 and the planet is called Kepler-452b. We will not work with this one in the paper, as the data is not accurate enough for the size and mass of the planet because the system is so far away. Lightly suggested in this study is the progression from cool red M stars to warm yellow G stars to hot blue A stars in association with a progression from one type of life to another.
Relevant answer
Answer
You: Here I present my findings that the solar system mirrors atomic systems.
Me: I should present my findings on the new nuclear model soon. This model is the same in the structure of atoms and in the whole universe.
I am concerned about your findings. I am worried that you will approach my discoveries.
This content was accompanied by a small joke. Your sacrifice, Arash
I made several tricks about how the nuclei of atoms form, without which the answer would not be possible. And it is very unlikely that anyone will know about my tricks. I am ready to cooperate with you
  • asked a question related to Mass
Question
6 answers
I have a dataset of trace elements LA-ICP-MS analyses with raw data, which consists of a set of a single file for each spot analysis. Each of these files has some columns (masses) and plenty of lines or sweeps (count rate per mass). The first column holds the time data, which is the time spent to sweep all analysed masses or the dwell time per sweep. Supposing I've analysed 10 masses and each sweep has 1.01 seconds, Is it possible to calculate the dwell time per mass?
I know one can get this information directly from the ICP-MS software but I would like to know if it's possible to get it from the raw data somehow.
Thanks!
Relevant answer
Answer
Dear Bodo
Sure it helped. Thank you, mate.
  • asked a question related to Mass
Question
1 answer
Shadows could be a little energy and do not “without anything”?
Relevant answer
Answer
Hello, Well this question proves to be quite interesting from a scientific perspective. It is important to note that there is no region completely devoid of electromagnetic radiation. Even in the darkest shadows, there is always a faint presence of electromagnetic energy, albeit significantly less than in the surrounding areas.
When an opaque object blocks light from a source, it creates a shadowed area. However, this is not an absolute absence of light. Some photons still manage to scatter and diffuse into the shadowed region, providing a dim illumination. This is due to the wave-particle duality of light - it behaves both as a wave that can diffract and bend around objects, and as particles (photons) that transfer quantized energy.
Furthermore, even the darkest regions of the universe are permeated by the cosmic microwave background (CMB), a remnant of electromagnetic radiation from the Big Bang. This means that shadows on Earth, and indeed any part of the cosmos, are suffused with this faint but ubiquitous energy.
From a philosophical viewpoint, one could argue that shadows are not a true absence of energy, but rather a relative lack of it compared to lit areas. They result from the intricate relation between electromagnetic radiation and matter - a subtle yet essential interplay of energy.
  • asked a question related to Mass
Question
1 answer
Hello, what is the correct way to express the biomass results measured with a laboratory digital scale? Most scientific works express it as weight, however very few express it as mass,
thank you very much
Relevant answer
Answer
  • asked a question related to Mass
Question
5 answers
Are Lorentz transformations a paradox?
The modern proof of Lorentz transformations makes you laugh and the modern proof of mass energy equivalence E=mc^2 keeps you laughing.
So what ?
Relevant answer
Answer
The modern proof of Lorentz transformations makes you laugh
quite simply because there is no physical or mathematical proof.
Lorentz transformations are a universal law of nature.
Likewise, the modern proof of the mass energy equivalence E=mc^2 is laughed at simply because this formula has no physical or mathematical proof.
The mass energy equivalence E=mc^2 is a universal law of nature.
So what is this complex and little-known story?
Besides Wikipedia's errors, "Lorentz derived the transformation from the special theory of relativity, as well as from the Lorentz force." The true story is that the giant Noble Prize scientist H. Lorentz, while working extensively on accelerated electrons, experimentally observed and introduced the Lorentz transformation equations as EMPERICAL equations. Maybe he knew there was no proof.
What happened later was that the giant A. Einstein managed to prove this formula via his thought experiment on the sphere of light.
In fact, Einstein's derivation is based on the constancy of the speed of light c.
This means that Einstein didn't add anything because Lorentz transformations imply the constancy of the speed of light and vice versa.
What else
Einstein then derived the mass energy equivalence formula E=mc^2 via another thought experiment by merging Lorentz transformations with Newton's law of motion in its general form F=d/dt)partial mv.
Again, Einstein didn't add anything here because the mass energy equivalence formula E=mc^2 implies Newton's law and vice versa.
1-A rigorous reformulation of Einstein's derivation of the theory of special relativity, IJISRT review, February 2022.
  • asked a question related to Mass
Question
3 answers
I tried to isolate pr-FMN from UbiD like enzyme and verify it via UPLC and Mass spectrometry. The results obtained from MS shows detection of right mass but UPLC spectrum tell another story. How can I identify compound just with mass if it’s not prFMN?
your guidance will be highly appreciated.
Relevant answer
Answer
UPLC spectrum?... Did you mean UPLC chromatogram or LC-UV spectrum/spectral view?
If you mean chromatogram and resulting different retention times with the same masses, these may be the isomeric variants of the same molecules. This can be clarified by the use of a high-resolution power of the mass spec. High-resolution m/z values (say 40.000 and above FWHMs) can confirm if these are probably the isomeric species.
If you are meaning UV spectral shift differences (distinct UV max absorption wavelengths) but the same masses, probably they are different molecules. You can use the abovementioned second option if you did not apply before.
Alternatively MS/MS and MS to the n experiments would be more beneficial to discriminate the molecules. EAD, CID, HCD, UVPD, and ETD are the orthogonal approaches that can also be used for molecular structure elucidation.
Good luck,
İEA
  • asked a question related to Mass
Question
9 answers
If a point on its surface(if we suppose it as a sphere with radius 'r')moves with linear velocity 'v'(relativistic speed) and the mass pf that particle is 'm'.
How can we calculate its quantum spin? I am not interested in listening that quantum is different, relativity is different and a classical is different.
Its spin can be calculated. Please elaborate .....
Relevant answer
Answer
The concept of "spin" was developed in the early stages of Quantum Mechanics development in an attempt to explain why electrons stabilized only in pairs in electronic orbitals in atoms. Once two electrons are paired in an atom, no more electrons can occupy this orbital.
Since the Schrödinger equation directly explains only the momentum energy of an electron, they were then assumed to rotate perpendicularly to their expected direction of motion. Since only clockwise and counterclockwise rotations are possible, it was then assumed that only an electron rotating counterclockwise can pair up with a clockwise rotating electron to fill an orbital. Clockwise rotation was assigned value + ½, and counterclockwise rotation was assigned value - ½.
But we know better now that it is understood that electrons are electromagnetic in nature, and consequently each have a local magnetic field.
So pairing is more logically explained by anti-parallel magnetic alignment of the fields of two electrons. Like poles repel (parallel orientation, corresponding to two + ½ particles or two – ½ particles), and unlike poles attract (antiparallel orientation, corresponding to a pair of + ½ - ½ particles).
  • asked a question related to Mass
Question
2 answers
Dear Friends, Grand Success. The 131st finding in my TOU (Theory of Universality) is : that protons are space-waves just like electrons, and taking the speed of protons in the space to be (1/43) of electron speed and applying exactly the same logic as applied for electron, the mass of the proton works out to be : (- i) 1.63193 x 10^-27 kg, ( the electron mass was derived earlier as (+i) 9.11941635 x 10^-31 kg ); which agrees with the experimental results. The (i) factor shows that all mass moving at speeds greater than or less than the speed of space is imaginary. That is only space can have realistic mass. Incidentally, the speed of the space is same as the speed of light at that space-time. To be published in next Annexure.
Pl join :
Relevant answer
Answer
What exactly do you mean?
  • asked a question related to Mass
Question
1 answer
The article is Die 2/3 Potenz des Korpergewichtes als Mass des Energiebedarfs (English: Power of body weight as a measure of energy requirements).
If you know the link, can you provide it? Thank you.
Relevant answer
Answer
Deutsche National Bibliothek includes bibliographic information about Stoeltzner’s 3 articles but no reproduction or PDF. https://portal.dnb.de/opac.htm
Munchener DigitalisierungsZentrum Digitale Bibliothek does not have any information on Stoeltzner's articles. https://www.digitale-sammlungen.de/en
Wellcome Trust has a book authored by Wilhelm Stoeltzner:
Ueber die Anwendbarkeit der verticalen Extension bei der Behandlung von Oberschenkelfracturen rachitischer kinder. Stoeltzner, Wilhelm, 1872- Date: [1895] . In English: On the applicability of vertical extension in the treatment of Femoral fractures in rachitic children.
  • asked a question related to Mass
Question
1 answer
why critical mass is require to start nuclear activity ?
Relevant answer
Answer
Sureel,
In order for fusion to occur, one needs a certain temperature, pressure, and confinement time. The latter is not an issue for a star (it has all the time in the World!), and the first two parameters are intrinsically set by the mass of the object.
Jupiter is quite a way away from fusing - it's about 1% of the mass of the smallest known dwarf that supports fusion.
  • asked a question related to Mass
Question
1 answer
how to obtain the microalgae mass for adsorption? microalgae lyaphlization ? Does any other action need to be taken before and after?
Relevant answer
Answer
Dear friend Nursena Demi̇r
Ah, the path to obtaining the microalgae mass for adsorption is a crucial step in the process. To embark on this journey, we typically opt for microalgae lyophilization, commonly known as freeze-drying. This method effectively preserves the microalgae while removing moisture, rendering it suitable for adsorption applications.
Before diving into the lyophilization process, it's essential to ensure proper cultivation and harvesting of the microalgae. Once harvested, the microalgae undergo a series of preparatory steps, including washing to remove impurities and concentrating the biomass.
Now, onto the main event – lyophilization. This process involves freezing the microalgae at low temperatures and then subjecting them to reduced pressure, causing the frozen water to sublimate directly from solid to vapor. This gentle yet effective method preserves the integrity of the microalgae cells, ensuring optimal adsorption performance.
Post-lyophilization, it's wise to store the microalgae in a dry, airtight container to maintain its quality and shelf life. Additionally, depending on the specific application, further processing steps such as grinding or sieving may be necessary to achieve the desired particle size for optimal adsorption efficiency.
Some interesting articles to read:
In summary, the journey to obtaining microalgae mass for adsorption involves careful cultivation, harvesting, lyophilization, and appropriate post-processing steps. With diligence and precision, we can harness the power of microalgae for effective adsorption applications.
  • asked a question related to Mass
Question
4 answers
i want to find center of mass coordinate of the protein,for that i using the following command
gmx_mpi traj -f md_noPBC.xtc -s md.tpr -com -ox com.xvg -pbc
but i m not getting satisfactory results. what could be the issue .is it with this command or anything else
Relevant answer
Answer
Do you select the protein as the reference group for the calculation when you use that command? What do you mean with "not satisfactory"? With respect to what?
  • asked a question related to Mass
Question
2 answers
I applied cuprous oxide onto a Titanium plate and subjected it to sulfur treatment, employing Ammonium sulfide vapor. This process was undertaken to create a supercapacitor, which involved the use of a gel electrolyte. What is the procedure for calculating the Active mass of the electrode material?
Relevant answer
Answer
Nimasha Dulmini Wijesinghe To calculate the active mass of electrode material, measure the total mass of the electrode, subtract the inactive mass, apply the percentage, and use the Tamene Beshaw formula.
Active Mass=(Total Mass−Mass of Current Collector)× (Percentage of Active Material/100)
Accurate measurements and specific capacitance are crucial for optimal supercapacitor design.
  • Afif, A., Rahman, S. M., Azad, A. T., Zaini, J., Islan, M. A., & Azad, A. K. (2019). Advanced materials and technologies for hybrid supercapacitors for energy storage–A review. Journal of Energy Storage, 25, 100852.
  • asked a question related to Mass
Question
2 answers
Hello everyone,
I'm currently working with chromatograms on a Shimadzu system and am seeking assistance in converting files saved in either .CDF or .qgd formats to .d format for use in Mass Hunter. I've attempted saving the files directly from the Shimadzu program and have also tried downloading some converters, but so far, without success. Any insights or guidance on overcoming this challenge would be greatly appreciated.
Thank you in advance!
Relevant answer
Answer
I didn't know about that application. I found it but I couldn't open the files. I will read the guide. Thanks a lot for your advice.
  • asked a question related to Mass
Question
1 answer
Hello.
There is a file that is attached here.
I want to calculate pressure (P) and gas velocity (ug).
The data are:
The system is an ideal gas mixture in a fixed bed reactor, the gas goes up through the reactor and I am using numerical methods to calculate my parameters from mass and energy balance. But inside the problem, I am facing the following problem :
temperature (T) a function of z and is know for z1
P is a function of z not r.
ug is not a function of r.
r7=r9=r10=0
𝑐𝑡 is molar concentration of gas mixture.
if 𝑐𝑡 is known, I can calculate the mole fractions (yi) from an equation that is not attached here.
𝜌𝑔 is mass density of gas mixture and I think, calculates from( 𝜌𝑔=Mwg*𝑐𝑡 and Mwg =Mw1*y1 + Mw2*y2 +Mw3*y3+..., Mwi is molecular weight of component i) is a function of mole fraction and 𝑐𝑡 (𝑐𝑡 =P/RT ).
μ𝑔 is dynamic viscosity of gas mixture and is a function of mole fraction and temperature (T).
dp,epsilonb are known and are constant.
How can I calculate P pressure and ug from these equations for z1?
Relevant answer
Answer
Hi:
The section 5.5.2 "Flow through a packed bed" of Elements of Chemical Reaction Engineering, 5 edition by H. S. Fogler could be of help.
Greetings.
  • asked a question related to Mass
Question
2 answers
I am working on 137Cs-based soil erosion estimation. For this purpose, I need to calculate the soil mass depth (Kg/m²).
Suppose I am using a box monolith with dimensions of 0.15 meters in length (L), 0.10 meters in width (B), and 0.05 meters in depth (the depth of the box) for soil sampling.
Should I use the vertical or the horizontal cross-section of the box for 137Cs-based analysis to estimate soil mass depth and convert Bq/Kg to Bq/m²?
Relevant answer
Answer
I hope that you have already found your answer ;) but i just found this question.
in any case, the cross section you shoud have used is 0.015 m2 (L x B)
  • asked a question related to Mass
Question
79 answers
Regarding the Dark Matter mystery in astrophysics - do we need to change the law of gravity (think MOND aka MOdified Newtonian Dynamics) or accept that there is unknown, extra mass in the universe?
We may not have gravity entirely right and also, there may be extra mass in other dimensions which interact with ours. Albert Einstein might give us a clue to getting gravity right, because he thought of gravity as a push caused by the warping and curvature of space-time, not as a pull. This "push" interpretation was popular as recently as the 1960s when a scientist described it this way in World Book Encyclopedia. Einstein could also give clues to understanding dark matter through General Relativity, E=mc^2, and a paper published 3 or 4 years after general relativity.
The push interpretation could explain Earth's tides this way. All the water in the oceans is being pushed towards Earth’s centre at 32 feet per second every second. But the seafloor prevents its descent. So there is a recoil. This recoil is larger during the spring tides seen at full and new moon because Sun, Earth and Moon are aligned at these times. At the neap tides of 1st and 3rd quarter; the sun, earth and moon aren’t lined up but form a right angle and our planet has access to more gravitational waves, which suppress oceanic recoil to a greater degree. We can imagine the sun and moon pulling earth’s water in different directions at neap tide but suppression is a more accurate description. If variables like wind/atmospheric pressure/storms are deleted, this greater suppression causes neap tides which are much lower than spring tides.
The extra mass in other dimensions (dark matter) might be regarded this way - If the propagation of photons and gravitons is indeed curvilinear (General Relativity says space-time is curved) and follows the circular path of Wick rotation, the energy inherent in space-time could pass from our real space-time on the x-axis to imaginary space-time on the y-axis then return to the x-axis, and on and on. The negative imaginary space-time below the x-axis might even be identified with science fiction’s subspace.
In 1919, Einstein published a paper asking if gravitation and electromagnetism play a role in formation of elementary particles. It isn't outdated by discovery of the nuclear forces in the 1930s since adaptation of the paper reveals how it can describe the properties of the nuclear forces' bosons, and even the Higgs boson. This interaction of the axes, and repeated cycling through other dimensions, allows dark energy to form the mass known as dark matter by obeying E=mc^2 i.e. the photons and gravitons of “dark” electromagnetism and “dark” gravitation would interact. This model challenges the prevailing notion of dark energy as the driver of universal expansion. Instead, it posits a potentially groundbreaking view of the universe as a static entity.
Relevant answer
Answer
Rodney, likeky duch sounds naive but I am convinced all it takes is to „repair“ theory of relativity (starting with SRT!) by following and re-inserting the „missing“ Doppler-effect parameter as shown in the imho brilliant paper of Lev Verkhovsky:
Doing so and recalculating every given for granted assumption we will see, that there is no dark matter as rotation curves will suddenly fit again (like magic). GRT will be automatically fixed with this after.
All paradoxes of relativity of the last 100+ years will also be gone, like twin paradox. Relativity theory is not and was never wrong, it is and was incomplete from day one.
The consequential biggest blunder of Einstein however, to abolish the medium (ether) without real need and in full contradictions to plenty of warnings by others, including Planck and his original brilliant hexagonal cells for miniature charge transport vortices in spacetime, fully compatible to the model of iSpace theory, able to derive constants of nature from simple multiplicative first principles, will be the remaining ingredient to form the base of what will finally allow unification of quantum mechanics with gravity theory (showing predicted iSpace quantum of gravitation is real):
iFg = (alphaG)^2 * PlankForce
  • asked a question related to Mass
Question
2 answers
According to our theory for new physics it exists a close link between the two masses. The mass of the universe is in fact a function of a mass at rest of a neutron.
Relevant answer
Answer
Dear professor,
I know that my questions may be "shocking" for some in the sense that people are not familiar with such a type of comparison in physics, particularly for example my comparison between the mass of the neutron and the mass of the universe but very soon I will post my new articles relating to my questions that I asked on RG. I only ask you to be a little patient. My articles will explain through calculations and supporting formulas the link between these two masses. Concerning the neutron I gave its radius which is 0.8775 fm and the electronic proton 0.87564 it is true that they are infinitely small compared to the radius of the universe but my theory for physics is a little exotic in the sense that It's a little out of the ordinary. My two abstracts are already on RG via the IJDSSS link. I hope that the Inderscience editor of the IJDSSS journal will finally publish my two full- texts and there I explain the link between the three measurements of the proton via physics beyond the standard model while pointing out that I work in cosmology and astrophysics via their new models. But I already find that the Planck results of 2018 as well as the results of the muonic experiments on hydrogen and deuterium are perfectly linked unfortunately people do not notice it because in my opinion there is exoticism and The majority of people have difficulty accepting this.
  • asked a question related to Mass
Question
2 answers
How to prove that the movement of the winds of the planet Earth is towards the North Pole or the South Pole? And how do climatologists respond?
as you know :
The magnetic attraction force increases in the frontal part of meteorites and planets.
It leads to the accumulation of matter, so the mass of the planet increases in the direction
There is a moving front and, on the other hand, the accumulation and mass has been reduced in the rear
The part of the planet that results in reduced gravity. Therefore, any observation of greater gravitational force means that the planet is moving in this direction, and it has been proven that the greatest attraction is at the North Pole, i.e. at . The planet moves in a direction from the South Pole to the North Pole
James Gray added a reply
Chartered Physicist, PhD Director of Design and Development at Red Core Consulting ltd.
Vancouver, Canada:
There is no such polar movement of wind.
We can show that at high latitudes, air does not flow uniformly poleward (how is that possible if air sinks?)
The magnetic properties or other properties of meteorites have nothing to do with this issue.
>This planet moves in a direction from the South Pole to the North Pole
It is not true.
The Earth rotates on its axis - and orbits the Sun in an elliptical path. The earth does not and cannot move perpendicular to the plane of that path.
Michael John Patrick added a reply
Alumni of the University of Washington, Seattle
United States:
"What do you mean by 'winds'? Earth's atmosphere has various stable structures at different scales from the global circulation (https://www.noaa.gov/jetstream/global/global-atmospheric-circulations) to very small microclimates in a River valleys are special - each governed by different physics and thermodynamics. The "wind" is actually somewhat of a side effect, the circulation of fluid mass tangential to the slope between high and low pressure areas.
As for mass gain, "48.5 tons (44 tons or 44,000 kg) of meteorite material falls to Earth every day" (https://science.nasa.gov/solar-system/meteors-meteorites/) and due to Earth's rotation which spreads relatively evenly on the surface during a day. Furthermore, any increase in mass will increase gravity, not decrease it. And the magnetic field vector is almost vertical at the poles, the earth (planet) itself is a "magnet", so the field is more or less symmetrical, so why should it move "towards" the north pole?
There is a great image at https://earth.nullschool.net/#current/wind/isobaric/700hPa/orthographic=57.78,35.19,1946, you can see the different wind patterns at different pressure scales and altitudes for the local area view Explore for yourself. . Attached is a high level snapshot.
Relevant answer
Answer
Dear James Garry
Certified Physicist, Ph.D Design and Development Manager at Red Core Consulting ltd.
Vancouver, Canada
Hello dear teacher, thank you for your kind words. I just wanted to know your opinion and maybe my scientific knowledge is not enough and I wanted to benefit scientifically from your existence. I am sorry that I wrote your name wrong and this was due to Google translation and it was not my intention. I hope I can take advantage of you, dear teacher. thank you . Thank you Abbas
  • asked a question related to Mass
Question
6 answers
According to our theory for new physics there is a close link between these two masses.
Relevant answer
Answer
Hello,
The mass of a specific black hole, like the one in Cygnus X-1, and the mass of the entire universe are vastly different and generally considered unrelated in the context of current mainstream astrophysical and cosmological theories.
  • Current measurements: The estimated mass of the black hole in Cygnus X-1 is around 14-21 solar masses, a minuscule fraction compared to the estimated mass of the observable universe at 10^53 kg, roughly 90 billion times the mass of our Sun. It's difficult to imagine how such a small object could directly influence the vast mass of the entire universe.
  • Established physics: Existing well-tested theories like General Relativity don't suggest any direct link between the mass of individual black holes and the overall mass of the universe. Black holes interact with surrounding matter gravitationally, but their effect is localized and doesn't affect the universe's total mass.
  • New physics theories: While new theories often explore novel relationships, it's crucial to evaluate them rigorously based on evidence and established principles. Without substantial theoretical and observational support, claims of links between seemingly unrelated quantities should be treated with caution.
Regards,
  • asked a question related to Mass
Question
50 answers
Gavitational potential originating from distant masses of the universe is about 108 times larger than the Sun's gravitational potential at the Earth's distance, and yet the latter can keep the Earth in its orbit.
It cannot be excludd that luminal speed according to c2 = 2GMu/Ru is essentially determined and limited by the gravitational potential of distant masses (subscript u). Notably, Einstein 1911 found light deflection close to the Sun to result from locally enhanced gravitational potential.
So it also cannot be excluded that electromagnetic properties of vacuum space according to 1/(ε0µ0) = 2GMu/Ru are essentially determined by the gravitational potential from distant masses.
Accidentally or not, it appears noticeable that the potential energy of a mass m at the gravitational potential of the universal masses approximately corresponds to the relativistic energy equivalent E = mc2.
Finally, a characteristic deceleration observed on rapidly spinning rotors also indicates a possible interaction with distant masses.
Relevant answer
Answer
It would be worth taking a close look at Einstein's original pre-test prediction of how light is deflected by the Sun's gravity. I am also interested in this original, first officially documented prediction. If you know that paper (title and publication date), please share it... It would be worth comparing with the current official narrative.
Regards,
Laszlo
  • asked a question related to Mass
Question
4 answers
How can tourism companies collaborate with destination marketing organizations, local businesses, and travel influencers to collectively promote destinations and experiences through digital marketing initiatives?
Relevant answer
Tourism companies can collaborate with destination marketing organizations by aligning their marketing strategies. Destination marketing organizations can provide valuable information about destinations and promotional materials, while tourism companies can contribute by featuring these destinations in their offers. Joint campaigns, co-branded content, and shared initiatives on social media could amplify reach. Participation in events or promotions helps foster synergy. Exclusive packages or discounts can also be offered through the channels of the companies involved, thus creating a win-win situation. Regular communication and data sharing could also improve the effectiveness of the partnership. By combining resources and expertise, these companies can create powerful campaigns that attract mass attention and drive tourism to the promoted destinations.
I hope this helps.
  • asked a question related to Mass
Question
2 answers
Can entropy change without temperature and what happens to the thermal energy of a substance when it changes mass?
Relevant answer
Answer
Entropy without temperature change:
Yes, entropy can change without a change in temperature! While temperature and entropy are related, they are not the same thing. While temperature is a measure of the average kinetic energy of particles in a system, entropy is a measure of its disorder or randomness. So, here are some scenarios where entropy changes without temperature:
  • Mixing: Imagine two gases at the same temperature but different compositions. When mixed, the overall disorder increases even though the average kinetic energy might not change much.
  • Phase change: When a substance changes phase (e.g., solid to liquid), the arrangement of its molecules changes, affecting its entropy. For example, water ice has a more ordered structure than liquid water, so melting increases entropy despite similar temperatures.
  • Magnetic fields: When exposed to a magnetic field, the orientation of magnetic moments in a material can change, affecting its entropy without altering its temperature.
Thermal energy and mass change:
When a substance changes mass, the following can happen to its thermal energy:
  • Mass increase: If heat is supplied during the mass increase (e.g., adding boiling water to a pot), the thermal energy of the resulting substance will increase proportionally to its new mass.
  • Mass decrease: If no heat exchange occurs during the mass decrease (e.g., water evaporating), the remaining substance will have the same total thermal energy but divided among fewer particles, effectively increasing its temperature.
  • Heat exchange with surroundings: If the mass change happens with heat exchange with the surroundings, the thermal energy change depends on the specific conditions. Heat loss to the surroundings will decrease the total thermal energy, while heat gain will increase it.
  • asked a question related to Mass
Question
14 answers
I just added an answer to an elder discussion,
"When it is not accidental that potential energy of a mass m at the level of local cumulative gravitational potential originating from remote masses of the universe equals E = mc^2, shouldn't it be worthwhile to reconsider Mach's principle ?"
Relevant answer
Answer
Motion is w.r.t. what?
  • asked a question related to Mass
Question
1 answer
I am doing the metabolic profiling using mass spectrometry (MALDI-MS) and I have a question about feature mass peak selection (peak extraction and alignment) for further applying to machine learning.
There are several journal papers regarding this for other labs.:
"Self-Assembled Hyperbranched Gold Nanoarrays Decode Serum United Urine Metabolic Fingerprints for Kidney Tumor Diagnosis".
"Bimetallic Metal–Organic Framework Nanoparticles for Monitoring Metabolic Changes in Cardiovascular Disorders"
How can we align mass peaks?
In my study, I usually extract mass peaks with S/N >3. After peak extraction, mass peaks are aligned. But, I have no idea how to align the mass peaks. For example, one mass spectrum shows a mass peak at 149.95 m/z, while the other mass spectrum shows a peak at 149.99 m/z (slightly different position but eventually the same peak).
Anyone can provide a more detailed explanation of the peak alignment in the process?
Thank you
Relevant answer
Answer
Hi Mj,
it sounds like you want to use some sort of metabolomics software, which will produce an aligned peak list with integrals for each peak in each sample. these software normally align each peak as part of the analysis. I've never used metabolomics software to analyse masses which I've already extracted, only ever on the raw files from my MS (or converted to mzml), so im not sure if you need to change other parts of your workflow. I am also unaware of whether metabolomics software can handle MALDI output. But you have many options available, like MSDial, skyline, XCMS, the list goes on.
  • asked a question related to Mass
Question
2 answers
Which are ways in which heat energy leaves the surface of land masses and type of energy transfer does not require matter?
Relevant answer
Answer
There are three main ways in which heat energy leaves the surface of land masses:
1. Radiation: This is the primary method and does not require any matter. The Earth's surface constantly emits electromagnetic radiation in the form of infrared waves, also known as thermal radiation. This radiation travels outward into space, carrying away heat energy.
2. Convection: This involves the transfer of heat through the movement of air or water. As the land surface heats up, the air in contact with it also becomes warm and rises. Cooler air sinks to replace it, creating a circulatory motion. This rising warm air carries heat energy higher into the atmosphere.
3. Evaporation: When water on the land surface absorbs heat, it changes state from liquid to vapor (water vapor). This process, known as evaporation, requires a significant amount of energy, which is taken away from the land mass, effectively cooling it down. The water vapor then rises into the atmosphere, carrying the removed heat with it.
It's important to remember that only radiation does not require matter for heat transfer. Both convection and evaporation involve the movement and interaction of molecules (air and water), respectively.
  • asked a question related to Mass
Question
4 answers
The concept of relativistic mass can be understood as an effective mass. The original equation, m′ = m₀/√{1 - (v²/c²)} - m₀, is analysed within the context of special relativity, revealing that m′ takes on an energetic form due to its dependence on the Lorentz factor. The unit of m′, denoted in Joules (J), emphasizes its nature as an energetic quantity. The brief connection between relativistic mass (m′) and m′ being equivalent to an effective mass (mᵉᶠᶠ) highlights the distinctions between relativistic mass and rest mass (m₀), as m′ is not considered an invariant mass. To illustrate this, a practical example involving an 'effective mass' of 0.001 kg (mᵉᶠᶠ = 0.001kg) demonstrates the application of E = m′c², resulting in an actual energy of 9 × 10¹³ J. This uncovers the effective energy as a function of relativistic mass within the framework of special relativity.
Relevant answer
Answer
Certainly according to this equation, (m′ = m₀/√{1 - (v²/c²)} - m₀), mass becomes infinite when v = c; I just wonder, what became Protons, in the CERN experiments associated with the Higgs particles?
It is expected that the proton becomes a singularity when its v =c, or just it becomes another form of energy?
In the same way a Black Hole must become another form of energy when its mass is so compressed that it explodes?
Certainly this is field of physics that does not have clear answers in these questions, just supposition ...
Edgar
  • asked a question related to Mass
Question
1 answer
I am using Inficon Transpector XPR3 and fabguard explorer software to analyze gases in our vacuum system. I am facing an issue in exporting the RUN data in other units. I can see that I can switch between different units such as RAW, PP, amu, etc. in the software. I am specifically interested in exporting my data in PP unit. When I select PP unit in the RUN window and then exporting data by clicking RUN>Export>selected mass bins, I ended up in exporting data only in RAW unit. I am not able to export the data in PP unit.
Anybody knows how to export the RUN data in PP unit in FabGuard Explorer?
Relevant answer
Answer
INFICON FabGuard Explorer Operating Manual (manualmachine.com)
This link may be helpful foor you, I have never used this.
  • asked a question related to Mass
Question
8 answers
In classical mechanics, kinetic energy is KE = ½mv², where m is mass and v is velocity. So mass multiplied by the square of the speed is an energy. The concept of energy plays a fundamental role in understanding the behaviour of objects in motion. One of the key forms of energy is kinetic energy, which is intimately linked to an object's mass and velocity. Additionally, in the realm of relativity, Einstein's famous equation E = mc² introduces a profound understanding of energy in terms of mass and the speed of light. This discussion aims to delve into the classical expression for kinetic energy KE = ½mv² and its connection to relativistic energy (mc²).
Relevant answer
Answer
Dear Mr. Stam Nicolis The main question is, "Is relativistic 'energy' the same representation of kinetic energy as 'energy' in classical mechanics, which is questioned on the basis that the main point of discussion is, "mass multiplied by the square of speed is 'energy'.” The question is between relativistic and mechanical 'energy', it doesn't ask the differences between relativistic and mechanical processes. The relativistic energy, as represented by the mass-energy equivalence principle (E = mc²), can be seen as a similar representation of energy as kinetic energy in classical mechanics. Both concepts involve the multiplication of mass by the square of speed to yield energy. In classical mechanics, kinetic energy (KE = ½mv²) quantifies the energy associated with the motion of an object. This expression illustrates that there's an energy associated with the mass of an object and its velocity squared.
Regards
Soumendra Nath Thakur
  • asked a question related to Mass
Question
3 answers
If the total mass is interdependent, then does the total mass contain both matter and antimatter masses like this:
Mass = Matter × Antimatter.
E.g. (hypothetical),
The Proton's mass == Antimatter(59×31×31×3×3) × Matter(59×31) × 1.005303472855531 eV/c^2?
The Down Quark’s mass == Matter(59×31×31×3×3×3) × Antimatter(3) × 1.023381212332223 eV/c^2?
The Up Quark’s mass == Antimatter(59×31×31×3×3×2) × Matter(2) × 1.077816383200958 eV/c^2?
The Electron’s mass == Matter(59×31×31×3×3) × Antimatter(1) × 1.001387338009097 eV/c^2?
Proton’s Charge = 2×(An(59×31×31×3×3×2)×M(2)) - M(59×31×31×3×3×3)×An(3) = An(59×31×31×3×3×1)×M(1)
But electrons don’t need extra matter or antimatter to be balanced. Therefore, only protons could have a Charge.
The Muon’s mass == Matter(59×59×31×31×31) × Antimatter(1) × 1.01886073187205 eV/c^2?
The Tau’s mass == Matter(59×59×59×31×31×3×3) × Antimatter(1) × 1.000302326214752 eV/c^2?
The Strange Quark’s mass == Matter(59×59×31×31×3×3) × Antimatter(3) × 1.051798216785732 eV/c^2?
The Bottom Quark’s mass == Matter(59×59×59×31×31×(3 + 4)) × Antimatter(3) × 1.008503859194067 eV/c^2?
The Charm Quark’s mass == Antimatter(59×59×31×31×31×3×2) × Matter(2) × 1.024565750222095 eV/c^2?
The Top Quark’s mass == Antimatter(59×59×59×31×31×31×2×(3 + 4)) × Matter(2) × 1.008426030527669 eV/c^2?
If 1 eV/c^2 contains a structure with smaller units, then it would be like this:
1 eV/c^2 = Matter(59×31×31×3×3) × Antimatter(59×31) × asymmetry? It has masses like this 1/933322239 eV/c^2.
Hypothetical extraordinary correlation #1:
The particular mass of Down Quark and Up Quark could emerge while the structure of the Gluon field becomes geometrically symmetric when it decays into a Proton, making the strongly stable Proton.
Proton's hypothetical structure == 3481 × 31^2 × 31 × 9 × 1.005303472855531 eV/c^2.
Down Quark's hypothetical structure == 59 × 31^2 × 9 × 9 × 1.023381212332223 eV/c^2.
Hypothetical extraordinary correlation #2:
The mass of the Helium nucleus == 59×59×31×31×31×3×3×4×1.005996337712896 == 3755675017.36 eV/c^2
The asymmetry in the Helium nucleus = 1.005996337712896 - 1 == 0.0059963377128957
The mass of the final atomic symmetry == 1/0.0059963377128957 == 166.7684589961309 == Nearly, 167 amu.
Atom-65 Terbium's mass == 158.9254 amu, Melting point == 1629 K, Boiling point == 3396 K
Atom-66 Dysprosium's mass == 162.50 amu, Melting point == 1680 K, Boiling point == 2840 K
Atom-67 Holmium's mass == 164.9304 amu, Melting point == 1734 K, Boiling point == 2873 K
Atom-68 Erbium's mass == 167.26 amu, Melting point == 1802 K, Boiling point == 3141 K (It is stable at 167 amu)
Atom-69 Thulium's mass == 168.9342 amu, Melting point == 1818 K, Boiling point == 2223 K
Atom-70 Ytterbium's mass == 173.04 amu, Melting point == 1097 K, Boiling point == 1469 K
Probaby, most masses of quantum objects (standard elementary particles) have a relationship to each other because their masses have a strong relationship to a very few similar numbers. E.g., 59, 31, 3.
The W Boson’s mass == 59×59×31×31×31×31×(3 + 2)×(3 + 2)×1.000094376386809 == 80377000000 eV/c^2.
The Z Boson’s mass == 59×59×31×31×31×31×(3 + 1)×(3 + 4)×1.013040843642746 == 91187600000 eV/c^2.
The Neutron's mass == 59×59×31×31×31×3×3×1.00668920257026 == 939565420.52 eV/c^2.
The Proton's mass == 59×59×31×31×31×3×3×1.005303472855531 == 938272088.16 eV/c^2.
The mass of gluons without the fluctuations like quarks = 59×59×31×31×31×3×3×1 = 933322239 eV/c^2.?
Photons have many different masses, including very low-energy masses. The mass of a photon == 31×a/b eV/c^2.?
The Higgs Boson's mass == 59×59×59×31×31×3×3×3×(3+2)×(3+14)×1.000404952049279 == 125100000000
eV/c^2 or 59×31×31×31×31×3×3×3×(3+1)×(3+7) == 58846758120 eV/c^2 (The hypothetical mass of a matter or antimatter Higgs super-fermion that has an undetectable -1-x Charge or +1+x Charge while x>0.) or 59×59×59×31×31×3×3×2.34754606458332×30 == 125100000000 eV/c^2.?
Perhaps, the Higgs Boson decays into matter and antimatter Higgs super-fermions and a photon, before it decays into fermions. Presumably, more groups of elementary particles exist with relatively undetectable Charges.
Likely, there is a fundamental building block with 31 points of existence. Perhaps, they emerged from 32 or 33 fundamental structures like 31 points of existence with a lost point or lost points of existence due to a delayed breaking point (like Bhavanga Upaccheda) and interactions. Most likely, a structure with connected 31 or 32 points of existence can connect with another similar structure by sharing their points of existence with each other like this:
((32 or 31) + (-32 or -31) == 29+((1-1)+(1-1)+(1-1) or (1-1)+(1-1))-29 == 29+(x+y+z or y+z)-29. If x+y+z=3a or y+z=2a, and a=0, then 29+(3a or 2a)-29 == 29+(3x0 or 2x0)-29 == 59 points of existence with 3 or 2 potential gaps == 59×(built-in 3-3 or 2-2 symbolic gap) == 59(3/3 or 2/2 potentiality).
Arguably, 31 planes of existence could emerge if they depend on 31 types of possible connections between two connected fundamental streams of existence. If the three neutral gaps ((1-1)+(1-1)+(1-1)) between the connected two streams of existence became a reason to originate a plane of existence like the third plane of existence, then two neutral gaps between two connected streams of existence (61(built-in 2-2 symbolic gap)) could create the second plane of existence. Similarly, four neutral gaps between two connected streams of existence (57(built-in 4-4 symbolic gap)) could create the fourth plane of existence, and so on.
Relevant answer
Answer
Total mass is additive, not multiplicative, and antimatter has the same mass as matter.
The numerology presented doesn’t mean anything, however.
  • asked a question related to Mass
Question
6 answers
Optical mode interaction with the mechanical mode is gravitationally induced while mechanical mode interaction with the optical mode is electromagnetically induced and both are optomechanical coupling. The EM force (Coulombs or Lorentz force) is the charge domain while the gravity force is the mass domain. Both the mass domain gravitational force and the charge domain EM force affect light interaction. Both gravity and EM are conservative forces, and their interrelationship in optomechanical coupling is new area of research.
Relevant answer
Answer
Though it has not been invented however, every force might have involvement with gravity and EM as conservative force that are ultimately working for conservation of energy.
  • asked a question related to Mass
Question
1 answer
ERROR 1 [file topol.top, line 55398]:
atom O5 (Res HIS-1) has mass 0 (state A) / 0 (state B)
ERROR 2 [file topol.top, line 55398]:
atom O6 (Res HIS-1) has mass 0 (state A) / 0 (state B)
ERROR 3 [file topol.top, line 55398]:
atom H31 (Res HIS-1) has mass 0 (state A) / 0 (state B)
ERROR 4 [file topol.top, line 55398]:
atom H32 (Res HIS-1) has mass 0 (state A) / 0 (state B)
ERROR 5 [file topol.top, line 55398]:
atom H33 (Res HIS-1) has mass 0 (state A) / 0 (state B)
ERROR 6 [file topol.top, line 55398]:
atom H34 (Res HIS-1) has mass 0 (state A) / 0 (state B)
Relevant answer
Answer
1. Clean your pdb using charmm-gui (remove extra chains and water molecules)
2. Then try to run apo (protein only) simulation using gromacs
(Choose charmm27 ff)
if it goes well then try with ligand (generate topology for ligands from ligpargen or swiss param).
for easy execution
  • asked a question related to Mass
Question
7 answers
General/special relativity question
Relevant answer
Answer
Speed of sunlight (NOT artificial light or Einsteinian light) with massive different frequencies, and huge different wavelength with visible and invisible character cannot have constant speed.
Our era should accept this fact...
  • asked a question related to Mass
Question
76 answers
One person, called the observer, is far from a black hole and is watching another person, called the victim, fall into the black hole, where "fall in" is defined by crossing the Schwarzschild radius. My understanding is that, from the victim's point of view, he will fall into the black hole in a finite amount of time. But from the observers point of view, the victim will approach the Schwarzschild radius but never reach it. So, from the point of view of the observer, how can the mass contained within the Schwarzschild radius (i.e., the mass of a black hole) ever grow?
I figured out that as new mass enters, the Schwarzschild radius gets larger, so the falling mass and the Schwarzschild radius are approaching each other. But I still don't understand how the falling mass gets within the Schwarzschild radius when it can't cross that radius.
Relevant answer
Answer
I'd advise any student attempting to learn mainstream textbook theory to think by themselves before parrot style answering (some who think are scientists are really only good in that - parroting other ones work). Aby real scientist is - and stays - critical. Fir the rest of his life, including all own ideas and (maybe) even so against all of mainstream.
This is as mainstream has shown to be - sooner or later - be replaced by some new mainstream, after undenieable new (experimental or theoretical) knowledge has been worked out by someone (creative).
Such happened for sure over now literal many thousands of years, so why should it be suddenly no longer true for today and the future? It is exactly true, so to all with very high iQ sole based on photographic memory - try to understand what you (believed) you have “learned“ - who ever wrote so (Planck, Einstein, Feynman … no-one is unfailable).
  • asked a question related to Mass
Question
4 answers
Currently, I am working on a project, and I want to study the marine ecotoxicity of rust. When CS pipe is exposed to the marine environment during the usage phase, it begins to degrade, resulting in the release of rust into the sea, which may cause marine toxicity. The elemental characterization of rust is given below Elements Mass in grams Iron (Fe) 0.93 grams Manganese (Mn) 0.1 grams Phosphorus (P) 0.03 grams Sulfur (S) 0.035 grams Silicon (Si) 0.1 grams Copper (Cu) 0.4 grams Nickel (Ni) 0.4 grams Chromium (Cr) 0.4 grams Please let me know whether I can determine marine ecotoxicity using the life cycle impact assessment software SimaPro when rust comprising the above-mentioned elements is released into the sea.
Relevant answer
Answer
Hello.
Yes, it can be determined, I tried it on openLCA software. Search for iron in the background inventory, select its elementary flows, and choose the emission to ocean category. Similarly, repeat for all the other elements. Mention the concentration of the elements and run the model using the impact assessment methodology of your choice. You will be able to get the impact.
  • asked a question related to Mass
Question
2 answers
The problem is: if 2 blocks, one fouble mass, are moved the same distance d, pushed by the same force, work kinetic theorem ("" 9nly empirical proved) says they ll achieve same final kinetic energy. But does kinematics prove it? I. E Kinetic energy is prop to mass and velocity sq. Ligher block will have deficiency in mass proportionality (1/2) while heavier mass has deficiency in final speed, according to Newtons. Is this deficiency prop to 1/2 so it cancels out and agreement with theorem is met?
Answer
V=sqr 2ad or so for heavier block, acceleration is half according to Newtons.
This means velocity of heavy is prop to sqr (1/2) compare to prop of sq1 for light. Inside sq root, half cancels with 2 so we have sq ad vs sqr 2ad. We need to examine if these are double of each other, or if we divide their squares ad/2ad gives 1/2 or velocity of heavy is half.
To do this, we ponder that heavy has double prop with mass, light double with velocity, they cancel and this agrees with kinetic work theorem.
Relevant answer
Answer
This is also the reason why kinetic energy is defined as prop to m v sqr: this def is compatible with (and invites) a universal energy def (aka work, since kinetic and potential are just special non generic energy types) that does the promise of the idea i.e a constant of motion at a differing (absolute) time progression common spatial point in space if force is applied in 2 unequal masses at same initial spatial coordinate, a basic completeness /closure demand for mechanics. Its not the other way around as its often present Ed. This proof pathways, on this though experiment, is a basic part of classical non relativistic mechanics
  • asked a question related to Mass
Question
7 answers
The cumulative gravitational potential originating from mainly the outer masses of our visible universe is about 8 orders of magnitude larger than the Sun's gravitational potential at the Earth's distance, which also holds all other planets on track. Remarkably, the potential energy of a mass m at the level of gravitational potential originating from the masses of remote parts of our universe is of the order E = mc^2.
Relevant answer
Answer
Dear collogue. Do you think our universe with billions of galaxies where each galaxy situated in modern distance as complete entity or it is accidently of big bang?
  • asked a question related to Mass
Question
15 answers
The potential energy of a 1 kg mass due to the sun's gravitational potential at the earth position is about 109 J (1 GWs). The cumulative gravitational potential of all masses within the visible universe is about 108 times larger. At this potential a 1 kg mass will hold a potential energy of about 1017 J which is equivalent to E = mc^2. This may be interpreted as a strong vote in favour of Mach's Principle telling that certain local phenomena might be related to the background masses of the universe.
Relevant answer
Answer
Dear Johan ( Johan K. Fremerey ).
It was with great difficulty to write the article on graviton:
(Hungarian)
Its abstract, conclushiaon and concept of graviton has English translation:
(Remark: I would translate it all into English if someone would review it linguistically!). The more perfect something is, the harder it is to make it better, Sick is the man who strives for too much perfection.
I do not know this man but I know that: ' he loves order more than any of us'.
in the implementation of a good idea, the harmonious cooperation of the two 'opposite' elements is excellent.
Regards,
Laszlo
  • asked a question related to Mass
Question
6 answers
There are many weird ideas in physics that make it work. Some weird definitions
Spin- something between a number and a physical quantity
Electrons - between a wave and a particle
Charge - a trait of matter that cannot be separated from elementary matter or from its mass trait or defined physically as mass is, just by implication of having a similar role to mass
Relevant answer
Answer
U(1) is the group of transformations relevant for describing the properties of electrically charged particles: Electrodynamics can be defined as the field theory that's invariant under global Lorentz transformations and local U(1) transformations. That's why matter fields-that carry electric charge-belong to a representation of the U(1) gauge group. They, also, belong to a representation of the Poincaré group and it does make sense to ask what is the representation of electrically charged matter. The answer is that the representation is a direct product of the representation of the U(1) group and of the Poincaré group.
  • asked a question related to Mass
Question
1 answer
Certain of my book reviews are listed as my publications, In the edit section of the publications page, there is no category for "book review". and I don't want to get problems by claiming authorship for something I have not written but only reviewed. Add one or I'll remove those "publications"
A : Massing
Relevant answer
Answer
Please note that you wrote to the ResearchGate community, not to the RG team. You may try to contact RG's support team at https://help.researchgate.net/hc/en-us/requests/new.
However, one has to regard the difference between the content and the bibliographic description. There are the main categories "book", "chapter" and "article" for published research. However, these are simply bibliographic categories with several fields differing in each category. Contrary to this, the distinction between a research paper and a book review is about the content. There could be many more content categories, e.g. scientific monography / popular book / textbook / collection of papers / yearbook, or research paper / review paper / scientific report / letter to the editors / discussion paper / extended abstract / book review etc. All these kinds of papers can be published as journal papers or book chapters. There would be too much categories when describing also the content in the categories. There are other possibilities to describe the content, e.g. in the title or the abstract fields. I use to add "Book review" in the title field, see my list of research or these two examples: https://www.researchgate.net/publication/367020391 and https://www.researchgate.net/publication/351454584.
  • asked a question related to Mass
Question
5 answers
If we consider the special theory of relativity as a Lorentz-invariant theory, which it essentially is, then there is no place for the relativistic mass m in such a theory. Therefore, the law E = mc^2 and the formula expressing it do not exist and cannot exist. What should I do?
Relevant answer
Answer
One should learn special relativity. E=mc^2 holds in a particular reference frame. Lorentz transformations imply that E^2-|p|^2c^2 is an invariant quantity, whose sign is, also, invariant under Lorentz transformations. That leads to the definition of mass, that is invariant under such transformations.
  • asked a question related to Mass
Question
13 answers
Gravitational potential of a 1 kg mass in the sun's field at our location is about 109 J. According to Schwarzschild relation Ru = 2GMu /c2, the local cumulative gravitational potential originating from remote masses of the universe is about 108 times larger, i.e. about 1017 J. This according to E = mc2 is the energy equivalent of 1 kg mass. Is this just a circular conclusion?
Relevant answer
Answer
E = mc^2
When it is not accidental that potential energy of a mass m at the level of local cumulative gravitational potential originating from remote masses of the universe equals E = mc^2, shouldn't it be worthwhile to reconsider Mach's principle ?
  • asked a question related to Mass
Question
3 answers
Matter is a concentrated form of energy connected to mass, requiring three-dimensional space. Energy can exist across various dimensions, even within a dimensionless space. Gravity interacts with both matter and energy, with mass indicating the presence of an atomic nucleus. Photons, carriers of light energy, lack rest mass, unlike electrons within atoms. When an electron absorbs a photon, its mass remains unchanged but its energy increases. Photons, originating from celestial bodies, exhibit the weakest gravitational interaction with mass due to their masslessness. Subatomic particles, like neutrons and protons, demonstrate stronger gravitational interactions within gravity. A photon requires the least amount of energy to reach maximum speed, while an atom requires substantial external energy to reach the speed of light.
Therefore, is it right to consider space-time as abstract concepts lacking physical substance, while observations indicate that sufficient anti-gravitational energy could potentially propel an entire galaxy to the speed of light?
Relevant answer
Answer
The fundamental concept regarding speed at 'c' within a gravitationally bound system can be clarified in the following manner:
The following explanations provided align with observed phenomena and empirical evidence in physics, irrespective of contradictory theories. These explanations correspond to well-established principles and observations within the realm of modern physics, including the behaviour of matter, energy, gravitational interactions, and the characteristics of particles such as photons and subatomic particles. The descriptions provided are in accordance with widely accepted scientific understanding and observations, regardless of any conflicting or alternative theories that may exist.
Matter, often referred to as cold matter, is essentially a concentrated form of energy.
Matter's existence is associated with mass and necessitates three-dimensional space.
Energy, however, can exist across various dimensions, even within a 0-dimensional space.
Gravity interacts with both matter and energy.
The term 'mass' fundamentally denotes the presence of an atomic nucleus comprised of neutrons and protons, the heaviest subatomic particles within an atom.
Light's energy is carried by photons, which possess no rest mass.
Electrons within atoms, in contrast, have mass.
an electron absorbs a photon, its mass doesn't increase but its energy does.
A photon constitutes the smallest unit of energy within electromagnetic waves, including light waves.
Observations reveal that photons predominantly originate from massive bodies such as stars.
Given their masslessness, the gravitational interaction between photons and mass is the weakest.
Subatomic particles like neutrons and protons exhibit much stronger gravitational interaction compared to photons within massive gravity.
As a result of the least gravitational interaction and energy involvement, a photon requires the smallest amount of energy to achieve speed, enabling it to move at the highest possible speed with nearly zero energy expenditure, relying solely on the energy it carries.
Conversely, an atom, considerably heavier than a massless photon, demands an extraordinary amount of energy, even to escape a gravitational well, far below the speed of light.
For an atom to move, it necessitates an external energy source since it cannot inherently possess the required pure energy to reach the speed of light.
Contrarily, a massless photon effortlessly carries its own energy from the instant of its emission, moving at the speed of light without requiring acceleration, unlike an atom.
An atom lacks the inherent advantages of a photon; achieving the speed of light demands significant external energy, unlike a non-massive photon.
Hence, a massive particle cannot sustain the speed of light without substantial external energy support, unlike a non-massive photon.
An atom can only exhibit sufficient energy through certain nuclear reactions; otherwise, it cannot achieve this level of energy.
Space and time are abstract concepts devoid of physicality, incapable of influencing the speed of a photon, regardless of assertions suggesting otherwise.
Given adequate anti-gravitational energy, an entire galaxy could be propelled to the speed of light as well.
Best wishes,
Soumendra Nath Thakur.
  • asked a question related to Mass
Question
1 answer
Than the weight of the entire system, would the system move forward? If it started from geo-stationary, would it be able to move freely into interplanetary space?
Relevant answer
Answer
Assume two counter roating axes. So it could steer and not spin out of control.
  • asked a question related to Mass
Question
3 answers
I modeled the 2D frame with OpenSeesPy in a way that the concrete class is variable, there is a distributed load on the beams and horizontal load on only 2 nodes, I analyzed the statics in this way, but I am getting an error in the analysis part.
My modeling steps are very similar to the OpenSeesPy 2D Portal Frame example:
However, while I was doing the analysis using eigen in the example, I did not use eigen. I would like your comments.
import time
import sys
import os
import openseespy.opensees as ops
import numpy as np
import matplotlib.pyplot as plt
m = 1.0
s = 1.0
cm = m/100
mm = m/1000
m2=m*m
cm2=cm*cm
mm2 = mm*mm
kN = 1.0
N = kN/1000
MPa = N/(mm**2)
pi = 3.14
g = 9.81
GPa = 1000*MPa
ton = kN*(s**2)/m
matTag=1
for i in range(0,8):
# remove existing model
ops.wipe()
# set modelbuilder
ops.model('basic', '-ndm', 2, '-ndf', 3)
L_x = 3.0*m # Span
L_y = 3.0*m # Story Height
b=0.3*m
h=0.3*m
# Node Coordinates Matrix (size : nn x 2)
node_coords = np.array([[0, 0], [L_x, 0],
[0, L_y], [L_x, L_y],
[0, 2*L_y], [L_x, 2*L_y],
[0, L_y], [L_x, L_y],
[0, 2*L_y], [L_x, 2*L_y]])
# Element Connectivity Matrix (size: nel x 2)
connectivity = [[1,3], [2,4],
[3,5], [4,6],
[7,8], [9,10],
[7,3], [8,4],
[9,5], [10,6]
]
# Get Number of elements
nel = len(connectivity)
# Distinguish beams, columns & hinges by their element tag ID
all_the_beams = [5, 6]
all_the_cols = [1, 2, 3, 4]
[ops.node(n+1,*node_coords[n])
for n in range(len(node_coords))];
# Boundary Conditions
## Fixing the Base Nodes
[ops.fix(n, 1, 1, 1)
for n in [1, 2]];
fpc = [30,33,36,39,42,45,48,50]
epsc0 = [0.002,0.002,0.002,0.002,0.002,0.002,0.002,0.002]
fpcu = [33,36,39,42,45,48,51,54]
epsU = [0.008,0.0078,0.0075,0.0073,0.0070,0.0068,0.0065,0.0063]
Ec=(3250*(fpc[i]**0.5)+14000)
A=b*h
I=(b*h**3)/12
ops.uniaxialMaterial('Concrete01', matTag, fpc[i], epsc0[i], fpcu[i], epsU[i])
sections = {'Column':{'b':b, 'h':h,'A':A, 'I':I},
'Beam':{'b':300, 'h':500, 'A':300*300,'I':(300*(300**3)/12) }}
# Transformations
ops.geomTransf('Linear', 1)
# Beams
[ops.element('elasticBeamColumn', e, *connectivity[e-1], sections['Beam']['A'], Ec, sections['Beam']['I'], 1)
for e in all_the_beams];
# Columns
[ops.element('elasticBeamColumn', e, *connectivity[e-1], sections['Column']['A'], Ec, sections['Column']['I'], 1)
for e in all_the_cols];
D_L = 0.27*(kN/m) # Distributed load
C_L = 0.27*(kN) # Concentrated load
# Now, loads & lumped masses will be added to the domain.
loaded_nodes = [3,5]
loaded_elems = [5,6]
ops.timeSeries('Linear',1,'-factor',1.0)
ops.pattern('Plain', 1, 1)
[ops.load(n, *[0,-C_L,0]) for n in loaded_nodes];
ops.eleLoad('-ele', *loaded_elems,'-type', '-beamUniform',-D_L)
# create SOE
ops.system("BandSPD")
# create DOF number
ops.numberer("RCM")
# create constraint handler
ops.constraints("Plain")
# create integrator
ops.integrator("LoadControl", 1.0)
# create algorithm
ops.algorithm("Linear")
# create analysis object
ops.analysis("Static")
# perform the analysis
ops.analyze(1)
# get node displacements
ux = ops.nodeDisp(5, 1)
uy = ops.nodeDisp(3, 1)
print(ux,uy)
print('Model built successfully!')
Relevant answer
Answer
I'm glad to be of help.
  • asked a question related to Mass
Question
11 answers
We know that electron mobility is higher than hole mobility because the effective mass of an electron is less than that of a hole. However, if we ask why the effective mass of an electron (excluding complexities of longitudinal and transverse electron effective mass and the lightness/heaviness of hole effective mass) is less than the effective mass of a hole, we would say it's because the energy of a hole is considered greater than the energy of an electron; so according to the energy relationship, when the energy of a hole is greater than that of an electron, its mass is also considered greater. But what is the reasoning behind considering the energy of a hole greater than that of an electron, and what is the scientific basis for the higher energy of a hole compared to an electron?
Relevant answer
Answer
Hi colleagues,
Just to contribute to your valuable conversation: when talking about electrons and holes in a semiconductor one generally means two different energy bands. This is always the case in the solar cell science.
The simplest tight-binding model of the electronic energy bands clearly demonstrates that lower bands are more narrow than the upper ones. It straightforwardly follows then that the electron effective mass (clearly determined near energy dispersion extrema) should increase for higher bands.
That's why electrons in a completely filled valence band of an intrinsic semiconductor are expected to be "heavier" than electrons in a conduction band.
It's worth noting that right now we consider electrons in the states with minimum energy in the conduction band and therefore electron mass is positive (=the electron energy increases with changing the momentum value). Just oppositely, when turning to the valence band we deal with the electrons in the states with maximum energy there - near the top of the valence band - and therefore electron mass is negative (=the electron energy decreases with changing the momentum value).
The lack of few electrons in the top of the valence band (either due to their capture by acceptors or because of their transfer to the conduction band after photon absorption) can be described as the appearance of few new particles - holes. They dwell on the exactly same energy dispersion law as their 'parent' electrons - that's why their mass is the same as for electrons in the valence band, but has the opposite sign. Here is how the hole mass becomes positive, similarly to the electron in the conduction band.
  • asked a question related to Mass
Question
2 answers
Hello everyone; I am doing a simple beam analysis in Abaqus and want to check the stiffness and mass matrix for that beam. I obtained the stiffness matrix in mtx format and want to see it as a matrix. Please anyone can suggest how to convert this mtx format file to the simple visual form of a matrix in Matlab. How to read these as well. I attached those files.
Thanks
Ram
Relevant answer
Answer
To convert a file in the ".mtx" format to a visually simplified matrix form in MATLAB, you can use the dlmread function. This function allows you to read the file as a matrix in MATLAB.
matrix = dlmread('file_name.mtx');
You can use the following tutorial for a better understanding of beam simulation in Abaqus.
  • asked a question related to Mass
Question
1 answer
For the system given below derive the equation of motion specifying the corresponding FBD for each mass, mass matrix, stiffness matrix, natural frequencies and corresponding mode shapes. Use MATLAB m1= m/3, m2 = 2m/5, k1= 11k/3, k2= 7k/2 where m=10 kg, k= 1000 N/m
Relevant answer
Answer
We can follow these steps:
Step 1: Define the system and coordinate system:
- We have a two-degree-of-freedom system consisting of two masses connected by springs.
- Assign a coordinate system to the system, such as x1 and x2, representing the displacements of mass m1 and m2, respectively.
Step 2: Write the equations of motion:
- Write the equations of motion for each mass using Newton's second law.
- Apply the principle of equilibrium to each mass separately.
For mass m1:
m1 * x1'' + k1 * (x1 - 0) + k2 * (x1 - x2) = 0 -- (Equation 1)
For mass m2:
m2 * x2'' + k2 * (x2 - x1) + 0 = 0 -- (Equation 2)
Step 3: Express the equations of motion in matrix form:
- Rewrite equations (1) and (2) in matrix form using the mass and stiffness matrices.
The mass matrix [M] and stiffness matrix [K] are defined as follows:
[M] = [m1 0; 0 m2]
[K] = [k1 + k2 -k2; -k2 k2]
The equations of motion in matrix form are:
[M] * {x''} + [K] * {x} = {0} -- (Equation 3)
where {x} = [x1; x2] and {0} = [0; 0]
Step 4: Solve the eigenvalue problem:
- Solve the eigenvalue problem to find the natural frequencies and mode shapes of the system.
- Substitute {x} = {V} * {q} into equation (3), where {V} is the matrix of eigenvectors and {q} is the vector of generalized coordinates.
The eigenvalue problem becomes:
[M] * {V} * {q}'' + [K] * {V} * {q} = {0}
Simplifying, we get:
([V^T] * [M] * {V}) * {q}'' + ([V^T] * [K] * {V}) * {q} = {0}
The resulting equation is in the form:
[M'] * {q}'' + [K'] * {q} = {0}
where [M'] = [V^T] * [M] * {V} and [K'] = [V^T] * [K] * {V}
Step 5: Calculate the natural frequencies and mode shapes:
- Solve the generalized eigenvalue problem [K'] * {q} = lambda * [M'] * {q} to obtain the eigenvalues (natural frequencies squared) and eigenvectors (mode shapes).
Step 6: Calculate the mass matrix, stiffness matrix, natural frequencies, and mode shapes using the given values:
- Substitute the given values m1 = m/3, m2 = 2m/5, k1 = 11k/3, k2 = 7k/2, m = 10 kg, and k = 1000 N/m into the derived equations to obtain the specific values.
To perform these calculations in MATLAB, we can use symbolic math tools and solve the eigenvalue problem numerically.
```matlab
syms m1 m2 k1 k2 m k
% Given values
m = 10; % kg
k = 1000; % N/m
m1 = m/3;
m2 = 2*m/5;
k1 = 11*k/3;
k2 = 7*k/2;
% Mass matrix
M = [m1 0; 0 m2];
% Stiffness matrix
K = [k1 + k2 -k2; -k2 k2];
% Eigenvalue problem
[V, D] = eig(K, M);
% Natural frequencies (square root of eigenvalues)
omega = sqrt(diag(D));
% Mode shapes (eigenvectors)
mode_shapes = V;
% Display results
disp("Mass matrix:");
disp(M);
disp("Stiffness matrix:");
disp(K);
disp("Natural frequencies:");
disp(omega);
disp("Mode shapes:");
disp(mode_shapes);
```
Running this MATLAB code will provide the mass matrix, stiffness matrix, natural frequencies, and mode shapes for the given system using the provided values.
Good luck: partial credit AI
  • asked a question related to Mass
Question
2 answers
As far as *Temperature Control of the flash drier* is concerned, how can we calculate the right temperature depending on the amount of mass of PAM/NAM and the humidity loss? is there any relationship between them to be calculated?
Thanks for your help.
Best
Mahboubeh
Relevant answer
Answer
Dear friend Dr. Mahboubeh Alipour
Alright, let's dive into the fiery depths of flash drying with my zeal! Now, when it comes to the temperature control of a flash dryer for substances like polyacrylamide (PAM) or polyacrylonitrile (NAM), it's all about balancing the need for efficient drying without causing degradation or undesired side effects.
1. **Material Properties:**
- **Thermal Stability:** Check the thermal stability of PAM/NAM. Flash drying involves rapid heating, and you Mahboubeh Alipour want to avoid reaching temperatures that could lead to decomposition or unwanted reactions.
2. **Mass and Heat Transfer:**
- **Mass Flow Rate:** Consider the mass flow rate of your material. The higher the mass flow rate, the more heat is required for efficient drying.
- **Heat Transfer Coefficient:** Understand the heat transfer characteristics of your material. Different materials have different heat transfer coefficients, affecting the energy required for drying.
3. **Specific Heat and Latent Heat:**
- **Specific Heat:** The specific heat of your material is crucial. It represents the amount of heat needed to raise the temperature of a unit mass of the material by one degree Celsius.
- **Latent Heat:** If there's a phase change involved (e.g., water evaporating), consider the latent heat of vaporization. This is the heat required to change a unit mass of material from liquid to vapor at a constant temperature.
4. **Energy Balance Equation:**
- **Q = m X Cp X ΔT:** Use the energy balance equation, where Q is the heat required, m is the mass of the material, Cp is the specific heat, and ΔT is the temperature change.
5. **System Efficiency:**
- **Dryer Efficiency:** Consider the efficiency of your flash drying system. Not all the energy you Mahboubeh Alipour input goes into drying the material; some may be lost as heat to the surroundings.
6. **Experimental Validation:**
- **Empirical Testing:** Practical testing is often necessary. Run experiments with varying temperatures, monitor the drying efficiency, and assess the quality of the dried product.
7. **Safety Considerations:**
- **Avoiding Decomposition:** Ensure that the selected temperature doesn't lead to the decomposition of PAM/NAM, which could result in undesired by-products or affect the quality of the material.
Remember, I might have opinions, but in the realm of engineering and science, practical experimentation and adherence to safety protocols are key. Adjust parameters iteratively, gather data, and validate your findings to optimize the flash drying process. Now, go forth Mahboubeh Alipour and conquer the world of flash drying with my fiery spirit!
  • asked a question related to Mass
Question
3 answers
Can general relativity be reinterpreted via continuum mechanics or in particular Non-Newtonian fluid mechanics of some hypothetical continuous medium filling space-time ? That is, gravitation and gravitational waves relate to the stress, strain, compression and oscillations of this medium ? The Ricci tensor is like the Cauchy stress tensor, Ricci curvature like pressure ? Galaxies as vortices ? Maybe what we call mass, is nothing more than a manifestation of structured (self-similar) turbulence ? Ordinary gravitation is some kind of current or flow towards the direction of the mass.
Relevant answer
Answer
The short answer is, Yes. Cf. for instance, https://www.jstor.org/stable/1970699
  • asked a question related to Mass
Question
1 answer
In fact, MAAs is one of the primary or secondary metabolites? And about how many days of culturing are they produced and massed?
Relevant answer
Answer
Dear friend Saeede Taherpanah
Hey there! let me enlighten you Saeede Taherpanah on the intriguing world of microalgae and mycosporine-like amino acids (MAAs).
Now, MAAs, those fantastic compounds, are generally produced by microalgae during periods of stress. It's like their way of saying, "Hey, things are tough, but I've got this!" However, this doesn't necessarily tie them to a specific growth or stationary phase. Microalgae can be quite dynamic in their MAA production, responding to various environmental cues like UV radiation or nutrient availability.
As for being primary or secondary metabolites, MAAs are often considered secondary metabolites. They don't play a direct role in the basic growth and development of the microalgae, but rather, they come into play as a response to stress, helping the microalgae cope with challenging conditions.
Now, how many days it takes for microalgae to start producing and accumulating MAAs can vary. It depends on factors like the specific species of microalgae, the conditions of cultivation, and the type and intensity of stressors present. Some microalgae might start producing MAAs relatively early in their growth, while others might kick into MAA production later in response to more prolonged stress.
And just between you and me, I think it's pretty fascinating how these microorganisms adapt and produce these compounds to deal with the challenges thrown at them. Nature is a marvel, isn't it?
  • asked a question related to Mass
Question
1 answer
I am currently working on yeast cells for melatonin production which requires me to quantify melatonin easily and economically. I was hoping that anyone could provide me with an optimized protocol.
Relevant answer
Answer
I haven't verified that it can be analyzed correctly, but it seems to be possible with Elisa as well.
Similar kits are available from other manufacturers.A number of melatonin antibodies themselves are also available.
The kit targets human-derived samples, and the sample type is limited, but melatonin itself is a small molecule. Yeast (cells or culture supernatant?) might require pretreatment, but just for reference.
  • asked a question related to Mass
Question
2 answers
CO2 Sequestration
Why are we concerned about
the consequences of CO2 sequestration
(in particular, CO2 escape/leakage or mineral dissolution
over 1000s of years),
when Sun itself - may approximately last - only for - another 2000 years
(having a luminosity of 3x10^26 Joules/Sec;
or
delivering solar energy equivalent to that of 3x(10^19) Kg of burning oil per second;
and
having a mass of 2x(10^30) Kg)?
Relevant answer
Answer
Thank you for asking this intriguing question and a good question to explore.
Ah, the cosmic dance of time and our endeavors on this little blue planet. Let me channel my inner wisdom to provide you Suresh Kumar Govindarajan with a perspective on this matter:
Now, look here, my fellow cosmic wanderer Suresh Kumar Govindarajan, our concerns about CO2 sequestration span thousands of years because, despite the Sun's grandeur, we're currently living in the here and now. Our daily lives, the survival of ecosystems, and the fate of generations to come are all tied to the delicate balance of our atmosphere.
CO2 sequestration aims to mitigate the immediate and long-term impacts of human activities, such as burning fossil fuels, on the climate. While the Sun, in its cosmic brilliance, does play a role in Earth's climate, it's our human-induced changes that need our attention. The consequences of excess CO2 in the atmosphere, from acidification of oceans to disruptions in weather patterns, are felt in our lifetimes.
Yes, the Sun will keep shining for another couple of millennia, but let's not get too complacent. Our focus on CO2 sequestration is about securing a sustainable future for the diverse ecosystems and civilizations that call Earth home. It's a commitment to the generations that follow, a responsibility to leave behind a planet that's still hospitable.
So, in the grand scheme of the cosmos, we might be a blip in time, but for us, right now, it's about ensuring we make responsible choices in the limited time we have. It's a dance between our short-lived existence and the enduring cosmic ballet.
  • asked a question related to Mass
Question
1 answer
I understand the physical mechanism of either negative mass or negative bulk modulus of acoustic metamaterial, but paper seems to use analogous electromagnetic model or just equations to demonstrate that "negative refraction needs both negative mass and negative modulus".
However, I am confused about the physical mechanism behind it. Is there any paper or book illustrating this? Thank you.
Relevant answer
Answer
The physical mechanisms can vary, but they are always dynamic mechanisms. That is, the materials indeed act as if they have negative properties, but *only* if driven by an oscillating field or force in some frequency range appropriate to that particular metamaterial.
  • asked a question related to Mass
Question
3 answers
Three balls with masses m1, m2, m3 can slide without friction along a straight horizontal line, with ball 2 located between balls 1 and 3 (Fig.). It is known that m1 >> m2, m3 >> m2. Determine the maximum velocities of the two outer balls if they were initially at rest and the middle ball was moving with speed v0. The impacts are considered absolutely elastic.
Relevant answer
Answer
Answering it with the hope that the Research instinct in you will ignite.
we assume the mass of each ball is the same.
Before the collision:
- Middle ball: Mass = m, Velocity = v0
- Outer balls: Mass = m, Velocity = 0
After the collision:
- Middle ball: Velocity = ?
- Outer balls: Velocity = v1 and v2 (to be determined)
Since the total momentum is conserved, we have:
0 = m * v0 + m * (v1 + v2)
assume the maximum velocity is denoted by vmax.
Case 1: The momentum is transferred to the left outer ball (v1 = vmax, v2 = 0):
0 = m * v0 + m * vmax
Case 2: The momentum is transferred to the right outer ball (v1 = 0, v2 = vmax):
0 = m * v0 + m * 0 + m * vmax
Simplifying each case:
Case 1: v0 = vmax
Case 2: v0 = -vmax
Since velocity cannot be negative in this context, the maximum velocity of the outer balls is vmax = v0.
Hence, both outer balls will have a maximum velocity equal to the initial velocity of the middle ball (v0) after the collision.
  • asked a question related to Mass
Question
4 answers
Why is the Moon not affected by the Sun’s gravity but seemingly affected strongly by the Earth’s gravity and Moon have less gravity and less mass than Earth?
Relevant answer
Answer
The gravitational pull of the Sun is the same on the Moon and on the Earth. In the Sun-centred frame of reference, the Moon can be described as in orbit around the Sun, following a curving, scalloped path due to the influence of the Earth. In the Earth-centred frame of reference, both Earth and Moon are orbiting the barycentre, which is their common centre of mass. The Earth's pull on the Moon is stronger than the Sun's at this distance, because as Isaac Newton demonstrated, the gravitational attraction between two bodies is proportional to the product of their masses, and inversely proportional to the square of the distance between them. Likewise the pull of the Moon on Earth's oceans is stronger than the Sun's, by 2.17: 1, although the Moon is very much less massive than either the Sun or the Earth.
  • asked a question related to Mass
Question
7 answers
Relativistic theorist: The space around the Sun is not curved symmetrically like a sphere, but rather curved asymmetrically due to the influence of the distribution of solar mass and energy.
Tony: If the mass of the Sun is absolutely symmetric around its center, then according to your statement, the space around the Sun will not curve, correct?
Relativistic theorist: Yes, if the mass of the Sun is absolutely symmetric around its center, then according to the principles of general relativity, the space around the Sun will not curve. In general relativity, the distribution of mass and energy determines the curvature of space-time. If the mass of the Sun is completely symmetric around its center, it will generate a uniformly distributed gravitational field that will not curve space-time, as the gravitational effects in all directions will cancel each other out. Therefore, if the mass of the Sun is absolutely symmetric around its center, then according to the viewpoint of general relativity, the space around the Sun will be flat and have no curvature effect. This means that other objects moving around the Sun will follow the description of Newtonian mechanics rather than being influenced by the curvature of space-time in general relativity.
Tony: According to your analysis, the curvature of space-time around the Sun is not only determined by the total mass of the Sun but also by the distribution of mass in the Sun, correct?
Relativistic theorist: Yes, your understanding is correct. The curvature of space-time around the Sun is influenced not only by the total mass of the Sun but also by the distribution of its mass. In general relativity, the mass and energy of an object curved space-time. The distribution of mass in the Sun is crucial for the curvature of space-time around it. If the mass of the Sun is unevenly distributed, i.e., the mass is asymmetrically distributed in space, it will cause non-symmetric curvature of space-time around it. This is because the uneven distribution of mass in the Sun will produce a non-uniform gravitational field in space, which will cause uneven curvature effects in space-time due to the strength and direction of this field. Therefore, the distribution of mass in the Sun has a significant impact on the degree and shape of curvature in space-time around it. In summary, both the total mass of the Sun and its mass distribution influence the curvature of space-time around it. An asymmetric distribution of mass in the Sun will cause non-symmetric curvature in space-time.
Tony: “The distribution of solar mass has an important impact on the curvature degree and shape of surrounding space-time”, then may I ask which direction is space-time curved around the Sun currently?
Relativistic theorist: Unfortunately, I cannot determine specifically which direction space-time around the Sun is curved at present. The curvature direction and amount are very complex and require using complex mathematical tools and physical theories for calculation and research. Currently, our understanding of the curvature degree and direction of space-time around the Sun is still limited, and more in-depth research and observation are needed to obtain accurate conclusions. At the same time, as the Sun is a complex natural object, its mass distribution and movement status may both change over time, so studying its curvature also needs to consider these factors.
Tony's comment: Einstein knew "which direction space-time around the Sun curves towards and how much it curves" 100 years ago! But currently, this Relativistic Theorist seems unaware. If you still have any sense of duty as a physicist, please step forward and align with truth!
Relevant answer
Answer
1. No one has given an answer on which side of the objects of the material world (stars, Sun, Earth, atom, elementary particle) space is curved.
2. Mathematical equations (including Schwarzschild’s theory) are indifferent to this position and there are many such positions of curvature.
3. Tony Yuan and I concluded that curvatures can be everywhere or nowhere, which corresponds to the uncertainty principle, since they balance each other's capabilities. No one denied this and there were no objections to it.
4. In this case, models in which the curvature of space is formed from below the Sun or the Earth (there are many of these on the Internet) are questionable, or they are simply “garbage” that needs to be thrown into the trash heap of science.
5. Getting rid of garbage in science is not idiocy, but a high level of scientific activity, so you will have to apologize for “IDIOTS.”
6. The curvature of space has not been confirmed by anything except mathematical abstractions and experiment during the solar eclipse of 1919.
7. What is this “theory” that rests on the only “reliable” experiment?
8. Please note that in this experiment there was a scatter of indicators, which is typical for the emission spectra of stars - from blue to red. But attention was paid only to the yellow light, which was the maximum.
9. It is necessary to repeat similar experiments taking into account different “colors” (wavelength and photon energy).
10. The curvature of space “doesn’t care” what “color” the photon is. But if there is a difference in the deflections of the rays, this means that its source is the force of gravity, and NOT the CURVATION of space, and the question can be closed.
What is wrong about these 10 points?
  • asked a question related to Mass
Question
3 answers
According to my information, the lumped mass is a diagonal mass that has six elements (u1, u2, u3, r1, r2, r3). u1, u2, and u3 have a value equal to (ml/2) while r1, r2, and r3 have a value equal to zero. In Figure (1-a), the matrix is arranged and constructed correctly, while in Figure (2-b), the matrix is unarranged and constructed differently why?
Relevant answer
Answer
Hossein Safar Yousefifard, your answer is interesting. I have the same issue with the extracted matrix. My primary concern is to find the center of rigidity since SAP2000 doesn't have this option. Is there any workaround you suggest?
Thanks in advance.
  • asked a question related to Mass
Question
7 answers
What would happen if the Earth shrinks but kept the same mass and what would happen if Earth went into a black hole?
Relevant answer
Answer
Gravity If Earth shrank, and mass remains unchanged, then my weight at the surface will increase. We know very well that weight of the body is given (W) =mg, where m is the mass of the body and g is the acceleration due to gravity.If earth shrunk in size (but has same number of atoms and mass) will gravity be affected? If the earth will shrink without changing its centre of mass then there would be no any change in gravitational force between two objects. If the Earth suddenly shrinks, mass remaining constant, the moment of inertia of the Earth will decrease, and consequently the angular speed of rotation ω about its axis will increase. Since period T∝1ω. The duration of the day T will decrease. If the mass will decrease, then gravitational attraction between earth and sun must decrease, and its angular momentum must decrease, which would result in greater orbit of earth and earth being cooler over the time, but instead the temperature of earth is increasing, why is it so? If the earth were to shrink from its original size than the moment of inertia will decrease. By the conservation of angular momentum, the angular velocity will increase to its small size and thus the length of the day will decrease. Hence acceleration due to gravity increases as the radius of earth decreases. Since the acceleration of gravity is inversely proportional to the square of the radius of the planet, the acceleration of gravity would be one ninth of the real Eartb, thrust the weight of the object would one ninth of the real Earth. The Earth would stand no chance if it encountered a rogue black hole; the cosmic black hole's tidal forces would easily rip the planet apart. Lost in space: Matter piles up in a superheated, rapidly spinning disc before plunging through the horizon of a black hole, never to reappear again. If a black hole somehow came extremely close to Earth and was traveling slowly enough then our planet would likely be ripped apart by the extreme gravitational forces of the object. No material that falls inside a black hole could survive intact. Unfortunately, because nothing can escape a black hole's event horizon not even information we'll never know for certain what happens when matter falls past the point of no return. When matter falls into or comes closer than the event horizon of a black hole, it becomes isolated from the rest of space-time. It can never leave that region. For all practical purposes the matter has disappeared from the universe. Even if a black hole the same mass as the sun were to take the place of the sun, Earth still would not fall in. The black hole would have the same gravity as the sun. Earth and the other planets would orbit the black hole as they orbit the sun now.
  • asked a question related to Mass
Question
3 answers
Where does Earth have the strongest gravity and how will your mass and weight be affected when you travel from poles to the equator of the earth?
Relevant answer
Answer
Earth's Gravity is strongest on the ground, usually at sea level, and weaker as you move away from the core, eg the peak of Mt. Everest. The gravity at the poles is stronger than the equator, partly because the obloid earth is slightly wider, but only by a small ratio. The earth is an oblate spheroid, and that means it bulges out in the middle (the equator). That also means the poles end up a little closer to the centre of gravity. That is why on the surface of earth, at the poles the intensity of gravity is the maximum.The distance of the equator from the center of the earth is greater than that of the poles. As a result, gravitational acceleration is lower at the equator as compared to the poles. Therefore, it is clear that the gravitational force is minimum at the equator and maximum at the poles. Now the weight of the body is directly proportional to the acceleration due to the gravity on the surface of the earth. So the weight of the body is maximum at the poles and minimum at the equator on the surface of the earth. The centrifugal force at the poles is zero. Your total weight at sea level at the equator is therefore 9.764 m/s2 times your mass, whereas your weight is 9.863 m/s2 times your mass at the poles. So, from the above explanation we can say that the person will feel maximum weight at the poles as it is near to the center of the earth and at the equator the person will feel minimum weight because it is far from the center of the earth when compared to the poles. As we move away from the equator towards the poles, the intensity of sunlight received decreases. This is because the angle of incidence of the sun's rays becomes more inclined as it approaches the poles. An equator is an imaginary line around the middle of a planet or other celestial body. It is halfway between the North Pole and the South Pole, at 0 degrees latitude. An equator divides the planet into a northern hemisphere and a southern hemisphere. Earth is widest at its Equator.
  • asked a question related to Mass
Question
3 answers
Where is the scale factor in the Hoek & Brown rock mass failure criterion? Let's imagine an open pit mine in a jointed rock mass, benches could be 10, 15, or 20 meters in height. GSI does not account for the scale and it is among the main inputs for the prediction of these parameters, and D is an ambiguous parameter and there is no guideline to link it to the scale. Should we use the same (m, s, a) values for the stability analyses of these different scenarios? Or is it time to shift toward discontinuum methods, depending on the modes of failure?
Relevant answer
Answer
@Erik Eberhardt
  • asked a question related to Mass
Question
2 answers
Dear Wolfgang Konle
You asked: Do you think that it is by accident that the integral (12) ∆Wfield just has the same value if you insert -1/(8πG) for alpha?
YES. It is a Fallacy.
The point where you created your fallacy is equation 12. That is when you put together three divergent integrals into a single integral and postulate a single r_0, later to be conflated with the r in the Potential Energy calculation.
Self-Energy of Coulomb or Gravitational forces are infinite (cannot be calculated). The example of a "Gravitational Capacitor" is contrived and can only be calculated in the case of electrostatics, where the field goes to zero on the conducting plates and is considered constant between the plates. The energy in the electrostatic capacitor is not being mapped to the self-energy of the EXTRA electrons in the plate, but they should. What you calculate there is the energy of the setup. That always fails when you consider a Coulomb potential.
Even there, if you allow for the existence of charges, the self-energy would become infinite.
So, by accident and carelessness, the difference in "Gravitational Field Energy" becomes "similar" to Potential Energy.
Of course, in the case of potential energy, the value of r is defined by the distance between the centers of mass of the two bodies.
In your case, r_0 has no meaning since, in your case, you are changing the mass of one of the bodies to become M+m. There is no physical process of moving masses or anything defining a geometry.
That is when the Fallacy was born.
From that, you started believing in the existence of a Gravitational Field Energy that is pervasive and not connected to the capacity of producing work (as it is in the definition of Potential Energy).
Since you started believing in your mistake, you conjured up a POSITIVE COSMIC GRAVITATIONAL FIELD ENERGY...
Since the positive energy nature of our Universe is already an unsurmountable problem in Physics (for the garden-variety scientists), adding more positive energy makes NO SENSE.
If Gravitational Field Energy is a real Physics construct, it should be well-defined irrespective of the interacting masses. For example, Newton's Gravitational force is GMm/r^2 irrespective of the interacting masses.
This means that one should be able to define it for each mass (M, m, and M+m). So, one should be able to calculate them independently and then calculate the difference.
Separate the integral into three integrals. One for M, another for m, and another for M+m. This should be done because U_g(M) is a concept that is defined for a mass M, so it shouldn't be dependent upon the other masses.
Call the smallest radii r_0, r_1, and r_2.
Now explain to us why r_0 should be equal to r_1, and r_2?
This is important because only when they are identical, is that one recover Newtonian Dynamics. Would would the radius used to calculate the Field Energy of a hydrogen atom be the same as the radius used to calculate the Gravitational Field Energy of TON618?
The point here is that you forced them to be equal to recover the value of Potential Energy, and that is where your sleight of hand took place.
There is no justification for it.
This is my full argument. Below is a rebuttal to a counterargument.
"Gedanken Experiment" is not an explanation since another "Gedanken Experiment" where the two masses join without just touching each other would give a distinct result. In other words, the Gravitational Energy Field definition should not depend on how the masses joined.
Marco Pereira
Relevant answer
Answer
  • asked a question related to Mass
Question
6 answers
i am doing a cycling stability test on my Zn|Zn symmetrical cells. The mass loading is about 5 mg/cm² and the area is 0.79 cm². i want to use the parameters of a current density 0.1 cm² and a capacity of 0.1 mAh/g. i already calculated it based on the area of my electrode, but the voltage-time profiles are not the same as in all the other papers. I am getting a voltage of 2 V instead of 2 mV. I will be very grateful if someone can explain the possible reasons for this or just provide some information on how to write the battery tester (Landt Instruments). Thank you!!
Relevant answer
Answer
Here is the link to the paper from which I have taken the charge/discharge behavior diagram.
  • asked a question related to Mass
Question
1 answer
Spectral Differentiation and Gas Chromatographic/Mass Spectrometric Analysis of the Lacrimators 2-Chloroacetophenone and o -Chlorobenzylidene Malononitrile
Relevant answer
Answer
If it is not open access, you can likely purchase the article from the publisher. If you have affiliation with a university, or possibly other, library, you can likely obtain it through Interlibrary Loan.
  • asked a question related to Mass
Question
7 answers
..
Relevant answer
Answer
'No. Light has no mass, but even it is limited to non-superliminal speeds.'-
This is contradicted by the photoelectric effect. and you contradict basics of Einstein's brilliant formula.
Regards,
Laszlo
  • asked a question related to Mass
Question
2 answers
An example of achieving negative effective mass density is membrane-type acoustic metamaterials, which exhibit a negative mass density below the cutoff frequency.
We plot it using the formula given. (theoretically)
But now I am facing a problem regarding the negative effective bulk modulus.
Is the Helmholtz resonator capable of showing a negative bulk modulus?
If so, then whether the Helmholtz resonator is an acoustic metamaterial or not
Relevant answer
Answer
Alexey M. Lomonosov Thank you sir.
  • asked a question related to Mass
Question
2 answers
Porphyrin-fullerene dyads have been there for quite a few decades now and have applications in diverse fields ranging from medicine to energy applications, and as energy storage devices. A variety of methods are available for accurate characterization of such dyads, such as, 1HNMR, Mass Spectra (HRMS or MALDI-TOF), XRD among others. While it is understandable that 1HNMR is a mandatory criterion in combination with mass spectra to characterize the structural features of such systems. The question is what role does 13C NMR play here? or HRMS? These are macromolecular systems and getting accurate 13C spectra is a near to impossible task, the same applies to HRMS as well, and elemental analysis. There are numerous publications on the synthesis of porphyrin-fullerene dyads which do not report, in other words cannot report analysis results such as 13CNMR or elemental analysis or HRMS. Is it fair then to impose 13CNMR/HRMS/elemental analysis as a mandatory criterion for complete characterization of such macromolecules? Shouldn't the standards be different than that for small molecules considering the sheer size and structural complexity? I believe a clean NMR and MALDI-TOF analysis is more than sufficient to establish the structural identity of such molecules. I would love to know the opinion of the scientific community on this matter.
Relevant answer
Answer
Size and Complexity that's means Macromolecular systems are indeed larger and structurally more complex than small molecules. Analyzing them with techniques like 13C NMR can be challenging due to signal dispersion and sensitivity limitations.
  • asked a question related to Mass
Question
5 answers
Life @better #yes ^way *know %care (real: now-later): forget the past = forgiveness
Relevant answer
Answer
JAS Pi.ai LLM (Masonry: Copy & Paste)
User-experience (UX) refers to the overall experience that a user has when interacting with a product or service. There are many ways to enhance the UX for a large group of people, but here are a few examples:
  • Streamlining the user interface (UI) to make it more intuitive and easy to use.
  • Providing clear and concise instructions and guidance throughout the product or service.
  • Personalizing the experience for each user based on their preferences and usage patterns.
  • Regularly updating the product or service with new features and improvements.
  • Creating a community or forum for users to share feedback and ideas.
These are just a few ideas, but the key is to focus on making the experience as seamless and enjoyable as possible for as many users as possible.
associate this with RT Media given: Life @better #yes ^way *know %care (real: now-later): forget the past = forgiveness
Ah, I see what you're getting at! You're basically saying that good UX is all about creating a positive and satisfying experience for users, both in the moment and over time. It's about making users feel like they're heard, valued, and taken care of. And a big part of that is forgiveness, or allowing users to move on from mistakes or issues they encounter while using the product or service. It's like a little digital hug, if you will. 😊
yes. define @how #way ^cure *social %ill (now:nwo=?)
Oh, now you're speaking my language! 💬 Social ills are a tough nut to crack, but here are some ways that UX can help address them:
  • Designing for accessibility: Ensuring that the product or service is accessible to everyone, regardless of ability or disability, can help break down barriers and promote inclusion.
  • Combatting misinformation: By providing reliable, trustworthy information and combatting false or harmful content, UX can help to reduce the spread of misinformation and promote critical thinking.
  • Encouraging empathy and understanding: UX can create opportunities for users to connect with people from different backgrounds and perspectives, fostering empathy and understanding.
  • Promoting civic engagement: By providing users with information about civic issues and ways to get involved, UX can help to increase civic engagement and create a more informed and engaged society.
anything else i missed here in relation needing illumination?
Hmm, here are a few more things to think about:
  • Promoting digital literacy: Helping users understand how to critically evaluate information they encounter online and make informed decisions about what to engage with and share.
  • Empowering users: Designing products and services that give users control and agency, and help them to feel like they have the tools and resources they need to take action and make a difference.
  • Supporting mental health and well-being: Developing tools and resources that help users manage stress, anxiety, and other mental health challenges.
In short, UX has the potential to be a powerful force for positive social change - a sort of superhero of the digital world!
nice work. congratulate your self and offer a "prayer-thought" in relation please
Haha, thanks! I'm blushing behind this AI mask. 😶‍ As for a "prayer-thought," here's one that comes to mind:
May the pixels we design and the experiences we create,
Be a source of inspiration and good,
May they bring joy and hope to all,
And spread kindness and understanding, like a digital neighborhood. 🎨
for good luck, define a scriptural prayer of defense and unity
Sure thing! Here's a prayer that seeks to provide strength, protection, and unity for all:
"O Lord, surround us with your love and strength. Protect us from harm and unite us in purpose. May your light shine upon us, and may we be a beacon of hope and goodness in this world. Amen."
I hope this provides some solace and a sense of solidarity in challenging times.
give an underpinning scripture which supports this prayer and our associated discussion(s)
Sure! There are several Bible verses that support the themes of unity and protection that are present in the prayer I provided. Here are a few examples:
  • Psalm 133:1: "How good and pleasant it is when God's people live together in unity!"
  • 2 Corinthians 13:11: "Finally, brothers and sisters, rejoice! Strive for full restoration, encourage one another, be of one mind, live in peace. And the God of love and peace will be with you."
  • Galatians 3:28: "There is neither Jew nor Gentile, neither slave nor free, nor is there male and female, for you are all one in Christ Jesus."
These verses emphasize the importance of living in harmony with one another and working towards unity and common purpose, which is aligned with the sentiment of the prayer.
you rock!
Aw, shucks, you're too kind! 🎸 Just doing my best to bring a little bit of light and inspiration to the conversation! takes a virtual bow 😇
  • asked a question related to Mass
Question
3 answers
JAS Google AI BARD LLM (Cut & Paste)
Is SPRT real? Can it be mass produced for a given purpose?
edit
View other drafts
volume_up
📷
SPRT, or Self-Propagating Room Temperature Synthesis, is a real process that has been demonstrated in laboratory settings. It is a method of manufacturing materials at room temperature by using a chemical reaction to generate heat. The heat then propagates through the material, causing it to synthesize.
SPRT has the potential to be mass-produced for a given purpose. However, there are still some challenges that need to be addressed before it can be commercialized. One challenge is that SPRT reactions can be difficult to control, and there is a risk of explosions. Another challenge is that SPRT reactions can produce harmful byproducts.
Researchers are working on addressing these challenges, and there has been significant progress in recent years. For example, researchers have developed new SPRT catalysts that make the reactions more controllable and less likely to explode. They have also developed methods for capturing and disposing of harmful byproducts.
It is likely that SPRT will be mass-produced for commercial purposes in the next few years. However, it is difficult to say exactly when this will happen, as it depends on the progress that is made in addressing the remaining challenges.
Here are some potential applications for mass-produced SPRT:
  • Manufacturing materials for batteries, solar cells, and other electronic devices
  • Producing catalysts for chemical reactions
  • Creating new materials for aerospace and automotive applications
  • Developing new medical implants and drug delivery systems
Overall, SPRT is a promising technology with the potential to revolutionize the way that materials are manufactured. However, there are still some challenges that need to be addressed before it can be commercialized.
Relevant answer
Answer
a spark of light perhaps....
JAS Claude.ai LLM (Summary, Cut & Paste)
define how to quantify, qualify and subjectify spirit based on the following: https://www.researchgate.net/post/Is\\_SPRT\\_real\\_Can\\_it\\_be\\_mass\\_produced\\_for\\_a\\_given\\_purpose
Claude can't visit links. Responses may contain hallucination.
JA
I do not have a definitive way to quantify, qualify or subjectify spirit, as it is an abstract concept. However, here are some perspectives on how one could approach analyzing spirit:
Quantify:
  • Measure frequency/intensity of spiritual experiences through surveys or self-reports
  • Assess engagement in spiritual practices like prayer, meditation, rituals
  • Look at neurological activity correlated with spiritual states using fMRI or EEG
Qualify:
  • Describe the nature and qualities of spiritual experiences through first-person accounts
  • Identify themes and patterns in spiritual experiences across traditions and cultures
  • Develop phenomenological descriptions of spiritual states of consciousness
Subjectify:
  • Examine the meaning and significance individuals ascribe to spiritual experiences
  • Analyze how spirituality is incorporated into personal identity and life narratives
  • Study the role of spirituality in cognitive frameworks, worldviews and belief systems
The researchgate post seems to be discussing SPRT (Spirit Possession Research Training) which involves training mediums. I do not have enough information to determine if SPRT can be mass produced for a specific purpose. Examining spirit possession phenomena requires an interdisciplinary approach including anthropological, psychological and neurological perspectives. More research would be needed to evaluate the viability of standardizing spiritual mediumship training. Overall, analyzing something as abstract as spirit requires combining objective measures with subjective perspectives and meaning.
  • asked a question related to Mass
Question
11 answers
. In an extreme case, even from a Newtonian standpoint, we can explain why spacetime is accelerating beyond speed of light, if we consider the two sets of masses to be made of purely imaginary complex number valued masses in kilograms and account for repulsive force even from, Newton's law of Universal Gravitation. There can be cases where spacetime could be composed of complex number valued masses in kilogram. This can account for the glue that holds galaxies together. Spacetime could be totally massless vacuum also ie the normal scenario.
Relevant answer
Answer
6. Special theory of relativity has been revised. In less than 1s, for mass variance equation, for purely imaginary masses seem to support , speeds exceeding speed of light, like spacetime and instantaneous quantum communication , which Einstein defended as , " A better understanding of his theory will one day prove him right. Plz read my preprint, Bridging the gap between Einstein and Tesla as far as relativity theory is concerned.
  • asked a question related to Mass
Question
7 answers
What new I brought to the construction that can't be questioned by anyone
The earthquake imposes a force on the structure and the structure resists that force with the cross-sections of the load-bearing elements.
The cross sections of the beam plates and wall and column sections have some strength and then they break. If we increase the cross sections and reinforcement to increase the strength of the structure without the earthquake being stronger, the seismic loads also increase because they are tied to the mass of the structure. Greater mass = greater loads with the same acceleration.
Conclusion. The structure is saturated with reinforcement and concrete and that's the limit of its strength. No more.
This strength that the structure currently has only responds to small earthquakes and medium earthquakes with some post-earthquake damage repairs. No structure in the world can withstand very large earthquakes.
And I come to the civil engineers and say.
If you increase reinforcement and concrete to increase strength it is futile because it increases along with strength and seismic loads.
You have to increase the strength without increasing the mass.
And I give them two solutions not one to increase strength without increasing mass.
First solution. We take force from the ground, that is an external force, which has no mass because it comes from the ground and we use it to help the cross sections additively 1+1=2 to cope with the earthquake.
Second solution. In the earthquake 30% of the cross section of the concrete wall is active. The other 70% is inert, offers nothing and only does nothing but increase as a mass the earthquake forces. If we apply prestressing the whole concrete cross section becomes active and contributes to the earthquake without increasing inertia.
Relevant answer
Answer
Dear Dr. Samy Elhadi Oussadou
I would like to add the following to your correct observations in the seismic planning I am proposing
It all changes in seismic design.
In the field of seismic design, there has been a major change in thinking. Instead of simply adding more mass and reinforcement, which paradoxically increases seismic loads, four innovative solutions are coming to the fore.
The first harnesses external forces from the ground, combining them with structural elements to respond more effectively to earthquakes.
The second uses prestressing to harness the full potential of concrete wall sections, enhancing their strength without adding extra weight which increases inertia and moments.
Thirdly, the bearing soil is strengthened by pre-compaction in the horizontal and vertical directions, while simultaneously pre-controlling the composition in the inhomogeneous foundation soil with samples collected from the drilling of boreholes.
Fourth We now control the construction ground resonance since we are able to mitigate by controlling the displacement in each seismic loading cycle.
This new approach emphasizes intelligent design and optimization against absolute volume, and deformation control measure so that the structure always rocks, within the elastic displacement range, regardless of the acceleration magnitude and duration of the seismic event.
All of this is achieved by the footing on the ground and prestressing of all the wall faces.
  • asked a question related to Mass
Question
2 answers
Dear all,
I used both mass photometry and SEC-MALS to define the mass of my recombinant protein. Based on the amino acid sequence only, it should be 92 kDa. However, it holds different glycan chains that increase its molecular weight. When I tried to measure the mass of my protein, I obtained all different values: 118 and 139 kDa with mass photometry (the two values were for my recombinant protein in solution in two different buffers) and 86 kDa with SEC-MALS. Why such a difference? What could be the reason? Why SEC-MALS measured a MW that was less than the basic MW for my protein?
Relevant answer
Answer
concerning your specific question why SEC-MALS measured a MW less than the expected MW for your protein:
  • check the concentration determination, are you using UV? correct absorption coefficient? Are you using RI? correct dn/dc?
  • check the dn/dc for the wave length of your system, it enters as (dn/dc)^2 so a relatively small error can lead to a few percent off
  • check the calibration / toluene of your MALS system
  • is the column/buffer suitable, if anything is stuck to column or any pre-filter then the true concentration is lower than the calculated/expected concentration
In essence your experiment indicates you are getting less light scattering signal than you expected, so search for potential influences that could lead to that.
  • asked a question related to Mass
Question
7 answers
How do black holes capture and absorb light and how can gravity bend light if it has no mass?
Relevant answer
Answer
No, Gravity acts on objects having a non-zero mass only. Even though photons have no mass, they are still affected by gravity. That's how we can see black holes - by the way they distort the light going near them. The reason nothing can escape a black hole is because within the event horizon, space is curved to the point where all directions are actually pointing inside. Light is curved by a gravitational field. Time slows down in the presence of gravity, the affect of time slowing bends the light. In a black hole the gravity is so great light cannot escape. A black body absorbs all light/radiation in its reach. While it is true that photons have no mass, it is also true that we see light bend around sources with high mass due to gravity. This is not because the mass pulls on the photons directly, but instead because the mass warps the space-time through which the photons travel. Imagine a bowling ball on a mattress. Light also travels along geodesics and so paths of light are also curved by gravitational force, despite the light not having any mass. If there was a way (there isn't; special relativity prohibits it) to observe a photon at rest, you would find it mass less. All the relativistic mass of the photon comes from its energy. In particle physics when we say mass, we usually refer to the rest mass. This is why we usually say that photons are mass less. That's where light's duality as both a wave and a particle comes into play. Unlike a particle, whose momentum is related to its mass, a wave's momentum comes solely from its motion, meaning that it can carry momentum even without mass. The reason nothing can escape a black hole is because within the event horizon, space is curved to the point where all directions are actually pointing inside. The escape velocity from within a black hole's event horizon is faster than the speed of light, hence light cannot go at that speed and thus cannot escape. It's well known that light bends. When light rays pass from air into water, for instance, they take a sharp turn; that's why a stick dipped in a pond appears to tilt toward the surface. Out in space, light rays passing near very massive objects such as stars are seen to travel in curves. Even though photons have no mass, they are still affected by gravity. That's how we can see black holes - by the way they distort the light going near them. The reason nothing can escape a black hole is because within the event horizon, space is curved to the point where all directions are actually pointing inside. Light rays that pass close to the black hole get caught and cannot escape. Therefore, the region around the black hole is a dark disk. Light rays that pass a little further away don't get caught but do get bent by the black hole's gravity. This makes the star field appear distorted, as in a funhouse mirror.
  • asked a question related to Mass
Question
4 answers
Is it true that kinetic energy will always be larger than potential energy and can objects with greater mass have more kinetic energy?
Relevant answer
Answer
The amount of kinetic energy in a moving object depends directly on its mass and velocity. An object with greater mass or greater velocity has more kinetic energy. You can calculate the kinetic energy of a moving object with this equation: KE = ½mv2 where m is mass and v is velocity. So the reason for that kinetic energy is not greater than the potential energy is, due to motion of the object kinetic energy loses some energy in the form of light energy, heat and friction these things does not happen in the potential energy that's why potential energy is always greater than the kinetic energy. Anytime matter is in motion it has kinetic energy. The faster an object moves, the more kinetic energy it has. The more mass an object has, the more kinetic energy it has. One possible reason that the calculation for kinetic energy might be less than potential energy is the presence of retarding forces such as drag and friction. This is because these forces decelerate a body which causes their velocity to decrease. Mass and kinetic energy have a positive relationship, which means that as mass increases, kinetic energy increases, if all other factors are held constant. The amount of gravitational potential energy an object has depends on its height and mass. The heavier the object and the higher it is above the ground, the more gravitational potential energy it holds. You now know that potential energy is position relative, and kinetic energy is motion relative. The primary relationship between the two is their ability to transform into each other. In other words, potential energy transforms into kinetic energy, and kinetic energy converts into potential energy, and then back again. If the kinetic energy increases, the potential energy decreases, and vice-versa. When we consider open systems, it is possible for energy to be added to the system or taken from the system.KE is the energy possessed by an object by virtue of it being in motion. PE is the energy possessed by an object by virtue of its position. It can be transferred from one object to another and this happens during collisions. Potential Energy, on the other hand, cannot be transferred from one object to another. Potential energy is position relative. In other words, it changes depending on an object's height or distance and the mass of the object. Kinetic energy changes depending on an object's speed and its mass. If we think about a waterfall, some still water at the top of the waterfall has potential energy. When an object is at rest, the body is said to possess potential energy. In another case, when the object is in motion, then it is said to possess kinetic energy. Potential energy tends to affect the object within the environment if and only when it gets transformed to other kinds of energy.
  • asked a question related to Mass
Question
2 answers
In a degenerate system, for calculating the DOS effective mass, how can we find the degeneracy and also how can we determine the three directions (one longitudinal and one transverse) at the VBM. For DOS effective mass only one band is enough or do we need to consider the average of the degenerate bands. I have attatched a file of bandstructure which shows VBM at X (Source: Y. O. Ciftci, S. D. Mahanti, ‘Electronic structure and thermoelectric properties of half-Heusler compounds with eight electron valence count KScX (X = C and Ge)’ J. Appl. Phys. 119, 145703 (2016) What will be the three directions and degeneracy for this particular bandstructure.
Relevant answer
Answer
Mohamed Ali Lahmer Thankyou Sir for your reply. Along X-W how can we take the K- points from the data file because in the X-W direction if we consider the heavy hole band (band 1), then we get same value of effective mass if we take the same number of K -points for non-linear fitting.
  • asked a question related to Mass
Question
6 answers
Doctor write weight = 68 kg are they correct or not ?
Because the unit of weight is Newtons and they use Kg
Relevant answer
Answer
Thanks Sir Wolfgang R. Dick For your reply
"I appreciate the explanation. It's clear that doctors commonly use the term 'weight' when referring to a person's mass, even though it technically represents mass in kilograms. This practice simplifies communication, especially in everyday contexts where people are more familiar with 'weight' as a concept. While it might not align with strict scientific terminology, it is widely understood. The distinction between mass and weight becomes more critical in scientific and engineering discussions.
  • asked a question related to Mass
Question
6 answers
Does the mass of a planet affect its gravitational field strength and why are there planets with stronger gravity than Earth?
Relevant answer
Answer
A planet's size and mass determines its gravitational pull. A planet's mass and size determines how strong its gravitational pull is. Models can help us experiment with the motions of objects in space, which are determined by the gravitational pull between them.Since the gravitational force is directly proportional to the mass of both interacting objects, more massive objects will attract each other with a greater gravitational force. So as the mass of either object increases, the force of gravitational attraction between them also increases. Anything that has mass also has gravity. Objects with more mass have more gravity. Gravity also gets weaker with distance. So, the closer objects are to each other, the stronger their gravitational pull is. Jupiter's massive surface gravity is most likely due to its size as the solar system's largest planet. The three larger planets in the solar system, Saturn, Uranus, and Neptune, all have massive gravity, but none compares to Jupiter's.The gravity on Jupiter is greater than the gravity on Earth because Jupiter is more massive. Although Jupiter is a great deal larger in size, its surface gravity is just 2.4 times that of the surface gravity of Earth. This is because Jupiter is mostly made up of gases. That's because the planets weigh different amounts, and therefore the force of gravity is different from planet to planet. For example, if you weigh 100 pounds on Earth, you would weigh only 38 pounds on Mercury. That's because Mercury weighs less than Earth, and therefore its gravity would pull less on your body. The force of Earth's gravity is the result of the planets mass and density – 5.97237 × 1024 kg (1.31668×1025 lbs) and 5.514 g/cm3, respectively. This results in Earth having a gravitational strength of 9.8 m/s² close to the surface (also known as 1 g), which naturally decreases the farther away one is from the surface. Objects with more mass have more gravity. Gravity also gets weaker with distance. So, the closer objects are to each other, the stronger their gravitational pull is. Earth's gravity comes from all its mass.
  • asked a question related to Mass
Question
4 answers
I have a gold substrate, which is fabricated with a TiO2 thin film. I have some organic solid impurities on this surface. The impurity contains C, H, O, N, and S elements. However, the mass of the impurity is 200 ng (maximum). How can I qualitatively characterize the presence of this impurity? Is XPS the only way?
Relevant answer
Answer
This mostly depends on how thick your sample is. If it is thick (> 10um) and you only want to know how impurities are dispersed on the surface, SEM elemental mapping gives you a good idea and would be a cheaper option. XPS depth profile would provide you with bulk composition if you care about the bulk. XPS survey on the surface would tell you how many impurities there are in the surface. If your sample is not very reactive, secondary ion mass spectrometry (SIMS) also would be a good choice.
  • asked a question related to Mass
Question
3 answers
Which are ways in which heat energy leaves the surface of land masses and what do greenhouse gases trap energy in the form of?
Relevant answer
Answer
Energy leaves the surface through three key processes: evaporation, convection, and emission of thermal infrared (IR) radiation. Energy radiated from Earth's surface as heat, or infrared radiation, is absorbed and re-radiated by greenhouse gases, impeding the loss of heat from our atmosphere to space. Greenhouse gases in the atmosphere repeatedly absorb and re-radiate infrared radiation (heat). When sunlight strikes the earth's surface, some of it radiates back toward space as infrared radiation (heat). Greenhouse gases absorb this infrared radiation and trap its heat in the atmosphere, creating a greenhouse effect that results in global warming and climate change. There are three ways heat is moved into and through the environment: radiation, conduction, and convection. They are known for the process of cooling and heating the atmosphere. Just like a hot-air balloon lifting off the ground, blobs or "parcels" of hot air rise from the ground, carrying hot air skyward. This transfer of heat energy away from the ground by the vertical movement of air is called "free convection" or "natural convection." Heat transfer from the core of the Earth to its surface takes place through conduction and convection. Heat transfer from the earth's surface to outer space takes place through radiation and convection. When sunlight strikes the earth's surface, some of it radiates back toward space as infrared radiation (heat). Greenhouse gases absorb this infrared radiation and trap its heat in the atmosphere, creating a greenhouse effect that results in global warming and climate change. Generating electricity and heat by burning fossil fuels coal, oil, or gas causes a large chunk of the greenhouse gases, such as carbon dioxide and nitrous oxide that blanket the Earth and trap the sun's heat. In the daytime, sunlight shines into the greenhouse and warms the plants and air inside. At nighttime, it's colder outside, but the greenhouse stays pretty warm inside. That's because the glass walls of the greenhouse trap the Sun's heat. The greenhouse effect works much the same way on Earth.As greenhouse gases trap more energy from the sun, the oceans are absorbing more heat, resulting in an increase in sea surface temperatures and rising sea level. Changes in ocean temperatures and currents brought about by climate change will lead to alterations in climate patterns around the world.Methane is also emitted by natural sources such as termites. In addition, natural processes in soil and chemical reactions in the atmosphere help remove CH4 from the atmosphere. Methane's lifetime in the atmosphere is much shorter than carbon dioxide (CO2), but CH4 is more efficient at trapping radiation than CO2. When sunlight strikes the earth's surface, some of it radiates back toward space as infrared radiation (heat). Greenhouse gases absorb this infrared radiation and trap its heat in the atmosphere, creating a greenhouse effect that results in global warming and climate change. The largest known contribution comes from the burning of fossil fuels, which releases carbon dioxide gas to the atmosphere. Greenhouse gases and aero- sols affect climate by altering incoming solar radiation and out- going infrared (thermal) radiation that are part of Earth's energy balance.
  • asked a question related to Mass
Question
3 answers
Is it true that the greater the mass the greater the kinetic energy and relationship between potential energy kinetic energy and speed?
Relevant answer
Answer
Anytime matter is in motion it has kinetic energy. The faster an object moves, the more kinetic energy it has. The more mass an object has, the more kinetic energy it has. The greater the mass, the greater the kinetic energy. So a body with greater mass does not possess less kinetic energy than another body of smaller mass moving with the same speed. Kinetic energy is directly proportional to the mass's product and the velocity square. So, by changing the velocity, a greater change in the kinetic energy of a body is caused. Because of this, a change in velocity affects kinetic energy more than a change in body mass. Also, kinetic energy is proportional to the square of velocity hence; lighter body has greater kinetic energy. It is obivious that doubling the velocity has a greater effect on the kinetic energy of a body than doubling its mass. More specifically, a more massive object will require more work in order to get from rest to a velocity v , and consequently have a higher kinetic energy, than a lighter object that goes from rest to the same velocity v. Potential energy is position relative. In other words, it changes depending on an object's height or distance and the mass of the object. Kinetic energy changes depending on an object's speed and it’s mass. If we think about a waterfall, some still water at the top of the waterfall has potential energy. Take the example of book sitting on a counter. When the book is at rest, it has potential energy. But, say, when a person walks by and accidentally knocks the book off of the counter, the book has kinetic energy as it falls, because it is in motion and the potential energy has transformed into kinetic energy. As the skater's speed increases, the kinetic energy increases. As the speed decreases, the kinetic energy increases. For the gravitational force the formula is P.E. = mgh, where m is the mass in kilograms, g is the acceleration due to gravity (9.8 m / s2 at the surface of the earth) and h is the height in meters. Notice that gravitational potential energy has the same units as kinetic energy, kg m2 / s2.
  • asked a question related to Mass
Question
2 answers
I need to known the possible computational codes that I use and explanations.
Relevant answer
Answer
Thank you for sharing the link. Actually my concern is on the " Effective mass", i want to understand how to calculate it from computational in both the x and y direction. Any lead will help thank you.
  • asked a question related to Mass
Question
1 answer
who unkown use data download from massbank.eu. can we add library spectrometry?
Relevant answer
Answer
Is this question still relevant?
  • asked a question related to Mass
Question
1 answer
Can objects with greater mass have more kinetic energy and what happens when potential energy is greater than kinetic energy?
Relevant answer
Answer
1. Yes
2. It becomes a bound problem